Constitutional Law SP21

अब Quizwiz के साथ अपने होमवर्क और परीक्षाओं को एस करें!

Hamdi v. Rumsfeld

In Hamdi v. Rumsfeld, it considered the lawsuit of Yasir Hamdi, an American citizen who had moved to Saudi Arabia and been captured in Afghanistan. Hamdi was imprisoned in a military jail in South Carolina without charge or the right to see a lawyer. The administration allowed him to return to Saudi Arabia on condition that he relinquish his American citizenship.

There was a hugely important case decided in 1990 concerning the constitutionality of statutes burdening religious practices. What was it and what did it do?

The case was Employment Division v. Smith, 494 U.S. 872 (1990), which you might know better as the "peyote" case. Before Smith, any governmental action substantially burdening a religious practice had to be necessary to achieve a compelling governmental interest that couldn't be served in a less restrictive way. Smith changed all that by holding that a valid and neutral statute of general applicability prohibiting certain types of actions that are not intended to burden religious beliefs but have the effect of burdening some religious beliefs will be upheld. Thus, the state no longer has to prove a "compelling interest." RELATED ISSUE: After Smith was decided, Congress tried to revive the compelling state interest test by passing the Religious Freedom Restoration Act (RFRA), which required the courts to follow the compelling state interest test whenever religion was substantially burdened. However, the Supreme Court ruled that RFRA was unconstitutional because the Court alone, not Congress, has the power to define the scope of the Fourteenth Amendment (and, for that matter, the scope of other amendments). Boerne v. Flores, 521 U.S. 507 (1997). This is true even though §5 of the Fourteenth Amendment gives Congress the "power to enforce, by appropriate legislation, the provisions" of that amendment. According to Boerne, there must be a "congruence and proportionality" between the injury to be prevented (state discrimination) and the means chosen by the state, and RFRA failed that test. One problem with the legislation was that Congress had accumulated no evidence of widespread religious discrimination to which it was reacting. N.B.: The federal government remains subject to RFRA. States may constitutionally enact state RFRAs adopting the compelling state interest test. Boerne v. Flores, 521 U.S. 507 (1997); Burwell v. Hobby Lobby, 573 U.S. ___ (2014). EXCEPTIONS: There are three: (1) strict scrutiny is still applied if the statute is a hybrid—that is, it affects not only religiously motivated actions but other constitutionally protected rights as well (e.g., speech, press, parental rights); (2) if the state conditions receipt of unemployment benefits on conduct prohibited by religious beliefs, strict scrutiny will apply; or (3) if a law is either not neutral or not general, strict scrutiny will apply. For the second exception, watch out for cases in which someone is denied unemployment benefits for refusing to do something that is contrary to his or her faith (e.g., working on Sundays). NOTE: Needless to say, almost every fact pattern involving a burden on religious practices is impacted by Smith. What we've done with the hypotheticals in this section is to take facts from older cases, but modify the reasoning as necessary to reflect Smith.

Abrams v. United States

The court upheld the Sedition Act of four Russian immigrants who had printed pamphlets denouncing American military intervention in the Russian Revolution. The nation's highest court thus endorsed the severe wartime restriction on free speech.

Boddie v. Connecticut

Voting

Cantwell v. Connecticut

Free Exercise Clause. govt and state cannot suppress the religious views of groups of people.(1940)

Wickard v. Filburn

Farmer grew more wheat than allowed because he was being funded by congress. Congress said they could regulate that because even though it was for his personal consumption, it would effect interstate commerce. Expansion of power of the Commerce Clause

US v. Lopez

Gun Free School Zones Act exceeded Congress' authority to regulate interstate commerce.

Planned Parenthood v. Casey

A 1992 case in which the Supreme Court loosened its standard for evaluating restrictions on abortion from one of "strict scrutiny" of any restraints on a "fundamental right" to one of "undue burden" that permits considerably more regulation.

Adarand Constructors v. Pena

A 1995 Supreme Court decision holding that federal programs that classify people by race, even for an ostensibly benign purpose such as expanding opportunities for minorities, should be presumed to be unconstitutional.

Sessions v. Morales-Santana

Archaic and Overbroad Generalizations versus "Real" Differences

Cruzan v. Director, Missouri Department of Health

Court found it acceptable to require "clear and convincing evidence" for removal of life support.

Skinner v. Oklahoma

Court found law providing sterilization of criminals unconstitutional because there was no logical basis for distinguishing between the perpetrators of the two crimes

Feiner v. New York

Court ruled that speech which "creates a clear and present danger of riot or disturbance" is not protected

Romer v. Evans

Determined that a state constitutional amendment in Colorado preventing protected status based upon homosexuality or bisexuality did not satisfy the Equal Protection Clause.

Rational Relations Test

Everything else is considered "nonsuspect" and is subject to the rational relation test—the classification need only be rationally related to a legitimate governmental interest. All economic and social welfare classifications fit into this category, including some that you might be tempted to consider suspect or quasi-suspect, like poverty, welfare, housing, education, government employment, and age. It states that for a state law to be valid, there need only be a set of facts imaginable that would make the law a reasonable method of achieving a legitimate governmental purpose. All economic and social legislation is subject to this test, as are classifications based on wealth, age, and mental retardation. In addition, if the burden on a fundamental right is not "undue" (so that the law is not subject to strict scrutiny), the rational relation test will apply. NOTE: It is difficult for a law to fail this test; however, watch for impermissible state motives, especially any indication that the state has acted with "animus" toward a group or a person.

Amendment VIII

Excessive bail shall not be required, nor excessive fines imposed, nor cruel and unusual punishments inflicted. -- Most often mentioned in the context of the death penalty, the Eighth Amendment prohibits cruel and unusual punishments, but also mentions "excessive fines" and bail. The "excessive fines" clause surfaces (among other places) in cases of civil and criminal forfeiture, for example when property is seized during a drug raid.

Moore v. City of East Cleveland

Facts: Appellant lived with her son and two grandsons Issue: Is the city ordinance that limited occupancy of any dwelling unit to members of the same "family" unconstitutional? Holding: Yes; ordinance violated DP of 14th Concurrence: Stevens: Ordinance constituted a taking of property without due process and without just compensation Dissent: To suggest that related persons have constitutional rights of association superior to those of unrelated person is to misunderstand the nature of the associational freedoms that the Constitution has been understood to protect Rationale: Court long recognized that freedom of personal choice in matters of marriage and family life is one of the liberties protected by DP

McCulloch v. Maryland

Maryland was trying to tax the national bank and Supreme Court ruled that federal law was stronger than the state law. Supreme Court ruling (1819) confirming the supremacy of national over state government.

Terminiello v. Chicago

Terminiello charged with inciting a riot because of his controversial speech (about racial groups) which upset many and caused fights to break out. Result: SC rules that the breach of peace ordinance is unconstitutional and that free speech may only be limited when there is clear and present danger that rises above public annoyance or unrest.

What type of cases may be brought under the original jurisdiction of the Supreme Court?

Under Article III, §2, Clause 2, the Supreme Court's original jurisdiction extends to all cases: Affecting ambassadors (and other public ministers and consuls); and Those cases in which a state is a party. However, original jurisdiction is most often exercised for controversies between two or more states, under 28 U.S.C. §1251. NOTE: Original jurisdiction can't be enlarged or restricted by Congress; however, lower federal courts could be granted concurrent jurisdiction by Congress. NOTE: Most cases brought under the original jurisdiction of the Supreme Court are between two states.

What types of jurisdiction do the federal courts have?

Under Article III, §2, federal jurisdiction extends to: Cases in which the parties' diversity of citizenship determines the jurisdiction: cases between citizens of different states, where the amount in controversy exceeds $75,000; Cases in which the subject matter determines the jurisdiction: cases that arise under the Constitution, federal law, or treaties ("federal question" jurisdiction); and Cases in which the identity of the parties determines the jurisdiction: Cases between a state and a citizen of another state; Cases in which the United States is a party (don't forget that the United States may only be sued with its consent, usually provided by federal statute); Cases between two or more states; or Cases between a state or its citizens and a foreign country or its subjects. NOTE: The requirement that the amount in controversy in diversity cases exceed $75,000 is statutory, not constitutional (28 U.S.C. §1332).

Plessy v. Ferguson

"separate but equal" doctrine supreme court upheld the constitutionally of jim crow laws a 1896 Supreme Court decision which legalized state ordered segregation so long as the facilities for blacks and whites were equal

Frontiero v. Richardson

14th amendment negates a federal law that allowed a woman in the armed forces to claim her husband as a dependant only if he depended on her for 50% + of his support, while a serviceman could claim dependent status for his wife regardless of actual dependency Gender discrimination is unconstitutional

Goldberg v. Kelly

14th amendment requires welfare recipients to have a notice/hearing before state can stop benefits

Lawrence v. Texas

A Texas law classifying consensual, adult homosexual intercourse as illegal sodomy violated the privacy and liberty of adults to engage in private intimate conduct under the 14th amendment.

When is a case moot?

A case becomes moot when the controversy is resolved after the case is filed, but before the outcome of the case has been decided. This can happen in a variety of ways: the law changes; the wrongful behavior in question has ended and couldn't reasonably be expected to recur; or a party has died or is no longer subject to the challenged statute (e.g., the statute covers minors and the party is no longer a minor). For a court to decide an issue, the issue can't be moot—i.e., it must be a controversy at all stages of review, not just when the complaint is filed. N.B.: Certain types of cases are reviewable even if the issue appears to be moot, because they involve important constitutional issues. First, cases "capable of repetition yet evading review" are an exception to the mootness bar (these include pregnancy and election cases). Second, cases in which a defendant has voluntarily, but not permanently, changed his conduct will be heard (the controversy is temporarily, not permanently, resolved, and thus not really moot). Finally, even though a major issue in a case becomes moot, if there are collateral consequences to one of the parties the case itself isn't moot (because the parties would still have a concrete stake in the outcome of the case, albeit a diminished one; an example would be a civil rights claim against a law firm for failing to make a woman a partner; if she is allowed to become a partner at any time during the suit, she'd still have a back pay, "collateral" issue). NOTE: Mootness and ripeness are both essentially timing problems; a "moot" case is brought too late and an "unripe" case too soon.

To establish a "case and controversy" as required by Article III, §2, what must a claimant show?

A claimant must establish that the matter in controversy: Is definite and concrete; Touches the legal relations of parties having adverse legal interests; and Involves a real and substantial controversy capable of specific relief through a decree of a conclusive character. Aetna Life Ins. Co. v. Haworth, 300 U.S. 227 (1937). MNEMONIC: CReDiT (Controversy; Relief possible; Definite; Touching) NOTE: The standing requirement is part of the "case or controversy" requirement, as is the prohibition against advisory opinions and unripe and moot claims.

What is a legislative veto?

A legislative veto occurs when Congress delegates power to an executive agency or the President, but retains the right to veto actions of the agency or the President. A legislative veto undermines the executive right to veto, and thus violates the Separation of Powers doctrine. Immigration & Naturalization Service v. Chadha, 462 U.S. 919 (1983). EXAMPLE: Congress grants the President the power to promulgate regulations concerning immigration that are to have the force of law. However, Congress requires the President to give it the proposed regulations before they take effect, so Congress can approve or disapprove them. This is an impermissible legislative veto.

Fisher v. University of Texas

Affirmative Action race can be an admissions factor in colleges

Strauder v. West Virginia

Allowed African Americans to have jury service.

The Spending Power

Congress's power to spend public revenue to meet broad public objectives.

Minnesota v. Clover Leaf Creamery

EQUAL PROTECTION METHODOLOGY: RATIONAL BASIS REVIEW

Kelo v. City of New London

Eminent domain case: Local governments may force the sale of private property and make way for private economic development when officials decide it would benefit the public.

Board of Regents v. Roth

Established teachers having property rights to their positions & due process protections

NFIB v. Sebelius

In 2012, the Supreme Court ruled that the federal government had the authority to require that individuals purchase health insurance because Congress had the authority to levy taxes. (p. 692)

Craig v. Boren

In this 1976 ruling, the Supreme Court established the "medium scrutiny" standard for determining gender discrimination.

Under what circumstances will a zoning ordinance be considered an improper racial classification?

Only when it can be proven that the purpose and effect of the ordinance is to exclude members of a racial minority from a residential area. Arlington Heights v. Metropolitan Housing Dev., 429 U.S. 252 (1977).

Board of Curators v. Horowitz

PROCEDURAL DUE PROCESS

Cleveland Board of Education v. Loudermill

PROCEDURAL DUE PROCESS

Memphis Light V. Craft

PROCEDURAL DUE PROCESS

North American Cold Storage v. Chicago

PROCEDURAL DUE PROCESS

Hunt v. Washington State Apple Advertising Commission

STATE REGULATION OF INTERSTATE COMMERCE

State v. Post

Slavery and the Constitution

What provisions of the Constitution provide Congress with the power to enforce constitutional rights?

The Necessary and Proper Clause and the Enabling Clauses of the Thirteenth, Fourteenth, and Fifteenth Amendments.

A state law directly contravenes a statute enacted by Congress. What clause in the Constitution would provide the answer for which of the two will remain valid?

The Supremacy Clause of Article VI. That clause states that: "This Constitution, and the Laws of the United States . . . shall be the supreme Law of the Land." In such cases, the state law is preempted by the federal law. NOTE: Most problems with the Supremacy Clause come about when there's no direct conflict between state and federal law; and the two merely address the same subject matter in different (but not explicitly inconsistent) ways. A direct conflict exists where one can't comply with both federal and state regulations, or the objectives of the two conflict. NOTE: Most Supremacy Clause problems involve the Commerce Clause.

Martin v. Hunter's Lessee

The Supreme Court can review the decisions of the highest state courts if they involve a federal law or the federal Constitution

Dandridge v. Williams

Travel

Missouri v. Holland

Treaties are the law of the land

District of Columbia v. Heller

U.S. Supreme Court case that upheld that the Second Amendment protects an individual's right to possess a firearm

Arizona v. United States

(3) Do federal immigration laws deny Arizona's law attempting to remove aliens from the state? Yes on most parts, however legally detained individuals may be checked for citizenship - police can't just stop anyone for citizen check.

When does a substantive due process problem arise?

When a governmental action intrudes on a right. If the right is both personal and "fundamental," the statute must meet the "strict scrutiny" test. If it's not both personal and fundamental, but rather, economic or social, it need only meet the "rational relation" test of validity.

Allied Structural Steel Co. v. Spannaus (1978)

declared a state law unconstitutional under the contracts clause because the state regulation of private pension funds altered significantly the contractual obligations of employers under previously existing private, unregulated pension funds

ANALYZING PROCEDURAL DUE PROCESS PROBLEMS

(1) Has a government action impaired a liberty or property interest? If not, procedural due process is not an issue. If so, go on to #2. (2) What procedural requirements must be satisfied? The two principal tools are notice and a hearing. However, they can range from a full, adversarial prior hearing to a promise of a prompt postdeprivation evidentiary hearing to a notification of charges and an opportunity to respond. The requirements are determined by weighing the importance of the property or liberty interest in question and the risk an erroneous deprivation in a particular procedure would create (considering the probable value of any additional safeguards) versus the importance to the government of the function in question, and the administrative and fiscal expense of a particular safeguard. RATIONALE FOR PROCEDURAL DUE PROCESS: To prevent inaccurate decisions. Where there is little risk of error, the individual will be entitled to correspondingly less procedural protection. NOTE: Although procedural due process also applies to the deprivation of life, that won't be discussed in these cards, as deprivation of life without procedural due process is an overwhelmingly obvious problem you can undoubtedly spot on your own.

Ex Parte McCardle

(1869) Allowed Congress to change the appellate jurisdiction of the Supreme Court Allowed Congress to change the appellate jurisdiction of the Supreme Court

Constitutional Modes of Interpretation (6)

(1) Historical: Associated with the theory of original intent or original understanding, under which constitutional and legal interpretation is limited to attempting to discern the original meaning of the words being construed as that meaning is revealed in the intentions of those who created the law or the constitutional provision in question. (2) Textual: Concerns whether the judiciary or another is bound by the text of the Constitution and the intentions revealed by that language, or whether it may go beyond the four corners of the constitutional document to ascertain the meaning, a dispute encumbered by the awkward constructions, interpretivism and noninterpretivism. (3) Structural: Using a structural argument, one seeks to infer structural rules from the relationships that the Constitution mandates. (4) Doctrinal: Proceed from the application of precedents (5) Ethical: Derive rules from those moral commitments of the American ethos that are reflected in the Constitution. (6) Prudential: Seeks to balance the costs and benefits of a particular rule.

Keep in mind that this is a very bare-bones guide and doesn't include every nuance of equal protection by any stretch of the imagination; however, it does give you a basic roadmap for examining equal protection. Ask these questions:

(1) Is the action involved a state action? If so, go on to #3. If not, go on to #2. (2) Is the action involved a federal action? If so, and it involves an area of unique federal power (e.g., power over immigration or war), it's subject to the rational relation test. Otherwise, federal actions are treated like state actions for equal protection purposes, but you should keep in mind that the Equal Protection Clause doesn't apply to the federal government; the Due Process Clause of the Fifth Amendment, which does bind the federal government, is said to include an equal protection guarantee. For federal actions, go on to #3. If not (i.e., the action is by neither the state nor the federal government, but by a private individual), there's no equal protection problem, because private individuals aren't bound by the equal protection guarantee. (3) Does the government action create a classification (e.g., men vs. women, resident vs. non-residents, legitimate vs. illegitimate children)? If so, go on to #4. If not, there's no equal protection problem; check for substantive due process. (4) Is the governmental discrimination intentional? Discriminatory effect in and of itself is insufficient to trigger strict or intermediate scrutiny; the statute must be facially discriminatory or intentionally unequally administered or have a discriminatory motive. A government act that only has a discriminatory effect is subject to the rational relation test, regardless of the basis of the classification. Go on to #5. (5) What's the basis of the classification? (a) Is the classification based on race? If so, the classification will be subjected to strict scrutiny: It must be necessary to promote a compelling state interest. This is now true even if the statute is affirmative action (see Richmond v. J. A. Croson Co., 488 U.S. 469 (1989)), and even if the affirmative action is being done by Congress rather than by a state (see Adarand Constructors, Inc. v. Pena, 515 U.S. 200 (1995)). Strict scrutiny will also be applied where the government is making a race-conscious decision to further operational needs, as for prison administration. Johnson v. California, 543 U.S. 499 (2005). NOTE: A race-based affirmative action measure might nonetheless be valid if it is very narrowly tailored and is designed to remedy the government entity's own past discrimination (not just discrimination in general). If the classification is not based on race, go on to (b). (b)Is the classification based on alienage? If so, go on to (c). If not, go on to (e). (c) Who's creating the (alienage-based) classification? If the federal government, it's subject to the rational relation test; it need only be rationally related to a legitimate governmental interest. If the state, go on to (d). (d) Does the (alienage-based) classification involve a "function at the heart of representative government"? If so, it's subject to the rational relation test; it need only be rationally related to a legitimate governmental interest. If not, it's subject to strict scrutiny. (e)Does the classification unduly burden a fundamental right (i.e., the right to vote; the right to have access to the courts; the right to travel; and (possibly) the right to be a political candidate)? If so, it's subject to strict scrutiny: It must be necessary to promote a compelling state interest. If not, go on to (f). (f) Is the classification based on legitimacy or gender? If so, it's subject to intermediate scrutiny: It must be substantially related to an important state interest. If not, go on to (g). (g) All other classifications are subject to the rational relation test— including virtually all classifications arising in "economic" and "social-welfare" legislation, and all classifications based on or involving age, mental retardation, poverty, welfare, housing, education, government employment, homosexuality, and access to the judicial process when no fundamental right is involved.

Whenever a statute burdens religion, ask these questions:

(1) Is the belief in question "religious" (i.e., does the person have a sincerely held belief that parallels a traditional religious belief—it needn't fit an established religion or sect)? If so, go on to #2. If not, it's not protected by the First Amendment. (2) Does the statute prohibit the belief itself, or merely conduct that somehow relates to the belief? If it's the belief itself, the statute is unconstitutional—the freedom to believe is absolute. If it's conduct associated with the belief, go on to #3. (3) Is the statute a hybrid—that is, does it affect not only religiously motivated actions but other constitutionally protected rights as well (e.g., speech, press, parental rights)? If so, it's subject to strict scrutiny—it must be necessary to achieve a compelling governmental interest, and there must be no less burdensome alternatives for achieving that goal. If not, go on to #4. (4) Is the state conditioning receipt of a benefit on conduct prohibited by religious beliefs (this is probably limited to unemployment compensation)? If so, the "compelling interest" test (see # 3) applies. If not, go on to #5. (5) Is the statute a valid and neutral (i.e., not directed at religious practices) law of general applicability? If so, it is constitutional even if it incidentally burdens religious practices. If not, go on to #6. (6) Is the statute either not neutral or not general? If it is either, the "compelling interest" test (see # 3) applies. Employment Division v. Smith, 494 U.S. 872 (1990).

When the government creates a classification based on race, what are the two compelling interests that may allow the classification to survive strict scrutiny?

(1) Remedying past discrimination; and (2) Enhancing diversity.

Name the rights that are "fundamental" for substantive due process purposes.

(1) The Right to Marry; (2) The Right to Custody of One's Children; (3) The Right to Keep the Family Together; (4) The Right to Control Upbringing of Children; (5) The Right to Procreate; (6) The Right to Purchase and Use Contraception; and (7) A Protected Liberty Interest for Competent Adults to Refuse Unwanted Medical Care. RELATED ISSUE: Although abortion was recognized as a fundamental right in Roe v. Wade, the government is now held to the undue burden standard of Casey; a "finding of an undue burden standard is a shorthand for the conclusion that a state regulation has the purpose or effect of placing a substantial obstacle in the path of a woman seeking an abortion of a nonviable fetus." N.B.: The Court has ruled that the right to privacy protects private consensual adult sexual activity, including homosexuality, without identifying this privacy as a fundamental right. The Court invalidated a homosexual sodomy statute in Lawrence v. Texas, 539 U.S. 558 (2003), using rational basis review. Same-sex marriage is also a constitutional right. Obergefell v. Hodges, 135 S. Ct. 2584 (2015). NOTE: Some textbooks treat the punitive damages cases and the Caperton decision about a judge's refusal to recuse from a case involving a campaign contributor as a new type of economic substantive due process.

Because almost all provisions of the first eight amendments apply to the states as well as the federal government via the Due Process Clause of the Fourteenth Amendment, it's easier to remember which provisions haven't been incorporated. There are only four that aren't incorporated:

(1) The Third Amendment right not to have soldiers quartered in one's home; (2) The Fifth Amendment guarantee of a grand jury indictment; (3) The Seventh Amendment guarantee of a jury trial in civil cases; and (4) The Eighth Amendment ban on excessive fines. N.B.: The Second Amendment right to bear arms was the most recent right incorporated. In 2008, the Supreme Court for the first time recognized that the Second Amendment protects an individual right to bear arms. Because the case establishing the right arose in the District of Columbia, the Court did not reach the question of incorporation. D.C. v. Heller, 128 S. Ct. 2783, 2813 n.23 (2008). The rationale was extended to incorporate the right of individuals to keep weapons for self-defense to the states in McDonald v. City of Chicago, 130 S. Ct. 3020 (2010). RELATED ISSUE: The Court does not always require the incorporated right to be applied to the federal and state governments in the same manner. Although under case law the Sixth Amendment requires that federal trials use a 12-member jury, states need not do so. States, unlike the federal government, may also allow non-unanimous jury verdicts in criminal trials.

US v. Darby

(1941) United States Supreme Court upheld the Fair Labor Standards Act of 1938, holding that the U.S. Congress had the power under the Commerce Clause to regulate employment conditions. The unanimous decision of the Court in this case overturned several long-standing precedents, notably Hammer v. Dagenhart

Katzenbach v. McClung

(1964) Because some food served in Ollie's Barbecue originated out of state, The U.S. Supreme Court held that Congress had the power, under the Commerce Clause, to ban racial segregation in the restaurant.

Nollan v. California Coastal Commission

(1987, 5th) The Court found that regulations must serve a substantial public purpose and that exactions are valid as long as the exaction and the project are reasonably related. The court also found that the California Coastal Commission's requirement to dedicate an easement for public beach access was not reasonable.

Gonzales v. Carhart

(2007) Upheld the Partial-Birth Abortion Ban Act of Roe v. Wade in 2003, but established stricter timeline limitations and limited procedure techniques on abortion

In general, when will governmental aid to a private religious school be valid?

(1) The aid must be for secular (non-religious) instruction. (2) Aid is more likely to be upheld where the decision about whether to use the aid in parochial or secular private school is made by the individual parent or student than where the state selects particular parochial schools to participate. (3) Aid to colleges is more likely to be upheld than elementary and secondary school grants, due to the lower risk of religious indoctrination. N.B.: Since about 1990, the Court has moved sharply in the direction of greater permissiveness on state aid to religious schools. For instance, the state can now take public school teachers and send them into parochial schools to teach secular subjects. Agostini v. Felton, 521 U.S. 203 (1997) (reversing a prior decision). EXAMPLES: Permissible aid: Lending secular schoolbooks to parochial schools; paying bus fare for parochial school students; providing in-parochial-school instruction in secular subjects by teachers who do most of their teaching in the public schools; furnishing funds for constructing buildings to be used exclusively for secular purposes; giving publicly funded tuition vouchers to be used at the private school (parochial or secular) of the student's choice, even if the vast majority of participants select parochial schools. Impermissible aid: Paying teacher salaries that are used directly for religious instruction in parochial schools; tuition reimbursement limited to parents of children who attend parochial schools.

What are the two sources of executive privilege?

(1) The doctrine of separation of powers; and (2) The inherent need to protect the confidentiality of executive communications. NOTE: Executive privilege is not a constitutional power. SCOPE: Executive privilege definitely encompasses military, diplomatic, and national security secrets—any other executive communications enjoy only a presumption of privilege. U.S. v. Nixon, 418 U.S. 683 (1974). NOTE: Since U.S. v. Nixon, courts have approved subpoenas of both current and former Presidents; Presidents have also voluntarily appeared in civil or criminal litigation or before congressional committees.

Lujan v. Defenders of Wildlife - 3 part Standing test

(1) The plaintiff must have suffered an "injury in fact," meaning that the injury is of a legally protected interest which is: (a) concrete and particularized and (b) actual or imminent; (2) There must be a causal connection between the injury and the conduct brought before the court; and (3) It must be likely, rather than speculative, that a favorable decision by the court will redress the injury

What three-pronged test must a state meet to show an appropriate time, place, and manner regulation of expression?

(1) The regulation must be neutral as to the content of the speech; (2) It must be narrowly tailored to serve a substantial governmental interest (note that it need not be the "least restrictive means available"; instead, it need only promote a substantial governmental interest that would be achieved less efficiently without the regulation); and (3) There must be alternative means of communication available. MNEMONIC: NAMES (Neutral; Alternative Means; Significant interest) Ward v. Rock Against Racism, 491 U.S. 781 (1989). N.B.: If a time, place, and manner regulation isn't content neutral, it can only be valid if either the speech in question falls into a non-protected category (e.g., obscenity) or the regulation satisfies strict scrutiny (i.e., it's necessary to achieve a compelling governmental interest and it's narrowly drawn to achieve that end).

PROCEDURAL DUE PROCESS REQUIREMENTS OF COMMON PROBLEM AREAS

(1) Welfare benefit termination: Notice + pretermination evidentiary hearing. Goldberg v. Kelly, 397 U.S. 254 (1970). (2) Disability benefit termination: Notice + promise of posttermination evidentiary hearing. Mathews v. Eldridge, 424 U.S. 319 (1976). (3) Public employment termination: If for exercising constitutional right (e.g., speech)—prior hearing. Perry v. Sindermann, 408 U.S. 593(1972). If not, depends on whether employee can only be terminated for cause. If employee can only be removed for "cause"—notice with opportunity to respond + posttermination evidentiary hearing. Arnett v. Kennedy, 416 U.S. 134 (1974). If employee needn't be removed for cause (i.e., serves at the will of public employer)—there is no property right so no procedural due process problem. (4) Public school suspension for disciplinary reasons: If student's presence in school is dangerous or disruptive—removal immediately followed by notice of charges + opportunity to explain. If not dangerous or disruptive, and suspension is for substantial length of time (e.g., ten days)—notice + explanation of charges + opportunity to explain must all precede removal. Goss v. Lopez, 419 U.S. 565 (1975). If not dangerous or disruptive and suspension is brief (e.g., two or three days)—ex parte review by school official suffices. Bethel School District v. Fraser, 478 U.S. 675 (1976). (5) Public school suspension for academic reasons: Prior notice + opportunity to respond (no hearing is required). Board of Curators v. Horowitz, 435 U.S. 78 (1978). (6) Prejudgment garnishment of wages by creditors: Notice + prior hearing. Sniadach v. Family Finance, 395 U.S. 337 (1969). (7) Prejudgment seizure of property (on installment sales contract) by creditors: No prior notice or hearing as long as: a) Creditor applies to judge (not clerk) for prejudgment seizure; b) Application includes affidavit based on personal knowledge of specific facts (not conclusions) justifying seizure; c) Creditor posts sufficient surety bond; d) Provision made for prompt postseizure hearing, where creditor must prove probable cause for seizure. Mitchell v. W.T. Grant, 416 U.S. 600 (1974). (8) Driver's license suspension: If probable cause to believe conditions of license have been violated—prompt postsuspension hearing. (9) Involuntary commitment to mental institution—Adult: Clear and convincing evidence of need to commit; notice and hearing except in exigent circumstances. Addington v. Texas, 441 U.S. 418 (1979). Child: Precommitment inquiry, but no prior adversary hearing if inquiry is sufficient. Parham v. J.R., 442 U.S. 584 (1979). (10) Termination of parental status: Prior hearing where parental unfitness must be proven by "clear and convincing evidence." Santosky v. Kramer, 455 U.S. 745 (1982). (11) Detention of "enemy combatants" who are citizens: U.S. citizens detained during war are entitled to due process, including at a minimum the right to counsel and the right to go before a neutral decision maker. Hamdi v. Rumsfeld, 542 U.S. 507 (2004).

When you're analyzing a question in which a state is being sued in federal court, there's a potential Eleventh Amendment problem (in that the Eleventh Amendment prohibits federal courts from exercising jurisdiction over certain kinds of suits against states). In fact patterns like these, ask the following questions:

(1) Who is the plaintiff? Eleventh Amendment applies to: All private plaintiffs. Eleventh Amendment doesn't apply to: Another state or the federal government as plaintiff (even when it's suing to protect private persons). (2) Is the state a defendant? (The Eleventh Amendment applies only to cases against states.) Yes: State government, agencies of the state. No: Political subdivisions, municipal corporations, counties, school boards, or state officers sued in their personal capacity (unless the action requests the officer be ordered to pay funds from state treasury for wrongful acts of the state or return property in the state's possession, in which case the state is the real party in interest). (3) What relief is sought? Eleventh Amendment applies to: Damages, past debt, retroactive relief of any type. Eleventh Amendment doesn't apply to: Injunctive or declarative relief or suits against state officials if suit seeks to force them to conform their conduct to federal (not state) law. (4) Has the state waived immunity? Yes: Express (in legislation itself) or implied (willfully engaging in federally regulated activities that are not essential state functions). No: Participation in a federal program (e.g., accepting federal funds in the absence of an explicit congressional requirement that doing so constitutes a waiver). (5) Does a congressional grant of jurisdiction apply? Federal statutes based on congressional power to enforce the Fourteenth Amendment (e.g., civil rights laws) are not subject to the Eleventh Amendment bar.

When legislation impairs a right, ask: Is the right a "fundamental personal" right? If yes, go to #1; if no, go to #2.

(1) Yes, the right is a "fundamental personal" right. These are the First Amendment rights (e.g., speech, press, religion, assembly, petition), interstate travel, voting, privacy (e.g., marriage, contraception, procreation, raising children, family interest), death, and fairness in the criminal process (e.g., right to counsel on first appeal). Here, the legislation must meet the "compelling state interest" test—it must be necessary to promote a compelling governmental interest. (2) No, the right is not a "fundamental personal" right. (Instead, the statute is merely regulating social or economic interests of lesser importance.) Here, the statute is only subject to the "rational relation" test—if there is a set of facts imaginable that would make the law a reasonable means to achieve a legitimate governmental purpose, the law is valid. Such laws include public health and safety measures and all kinds of business regulations, including trade practices, wage and hour regulations, price controls, and bans on discrimination against union (or non-union) personnel.

Calder v. Bull

(1798) SC upheld the constitutionality of a law designed to allow inheritance to those designated as beneficiaries under a will. Ex Post facto clause applies to criminal, not civil cases Justice Chase - unwritten limitations on federal power/ natural law principles and the constitution/"first principles of the social compact"/ The court examined its authority to review state legislature decisions. Justice Iredell - courts cannot strike down statutes based only upon principles of natural justice, there must be textual evidence. Affirmed the ability of the Supreme Court to review legislative acts, but based on something more than principles of natural justice

US v. Butler

(FDR) 1936 as a case in which the Supreme Court of the United States ruled that the processing taxes instituted under the 1933 Agricultural Adjustment Act were unconstitutional.

Railway Express Agency v. New York

The court held the state law, which allowed advertising on business vehicles but prohibited advertising on all other vehicles, was constitutional because the classification was rationally related to the legitimate state interest of public safety. Equal protection does not require that a classification eliminate all evil of the same type.

Nebbia v. New York

The court held the state law, which fixed milk prices, was constitutional and did not violate the due process clause. States are free to adopt whatever economic policy may be reasonably deemed to promote the public welfare so long as the laws are rationally related to the legitimate purpose.

Establishment Clause

-1st Amendment- Prohibits the government from making any law "respecting an establishment of religion." This clause not only forbids the government from establishing an official religion, but also prohibits government actions that unduly favor one religion over another. It also prohibits the government from unduly preferring religion over non-religion, or non-religion over religion.

Free Exercise Clause

-1st Amendment- The Free Exercise Clause reserves the right of American citizens to accept any religious belief and engage in religious rituals. Free-exercise clauses of state constitutions which protected religious "[o]pinion, expression of opinion, and practice were all expressly protected" by the Free Exercise Clause.[1] The Clause protects not just religious beliefs but actions made on behalf of those beliefs. More importantly, the wording of state constitutions suggest that "free exercise envisions religiously compelled exemptions from at least some generally applicable laws."[2] The Free Exercise Clause not only protects religious belief and expression; it also seems to allow for violation of laws, as long as that violation is made for religious reasons. In the terms of economic theory, the Free Exercise Clause promotes a free religious market by precluding taxation of religious activities by minority sects.[3]

Dred Scott v. Sanford

1857 Supreme Court decision that stated slaves were not citizens: slaves were property no matter where they were living and the Missouri Compromise unconstitutional Supreme Court case that decided US Congress did not have the power to prohibit slavery in federal territories and slaves, as private property, could not be taken away without due process - basically slaves would remain slaves in non-slave states and slaves could not sue because they were not citizens

Loving v. Virginia

1867 court case that declared all laws against interracial marriage unconstitutional

West Coast Hotel v. Parrish

1937 - Supreme Court upheld the Washington state minimum wage statute.

Korematsu v. US

1944 Supreme Court case where the Supreme Court upheld the order providing for the relocation of Japanese Americans. It was not until 1988 that Congress formally apologized and agreed to pay $20,000 2 each survivor

Dennis v. United States

1951, made it illegal to advocate or teach the overthrow of the government by force or belong to an organization with this objective. (upheld the Smith Act of 1940)

Brown v. Board of Education of Topeka

1954 - The Supreme Court overruled Plessy v. Ferguson, declared that racially segregated facilities are inherently unequal and ordered all public schools desegregated.

Heart of Atlanta Motel v. US

1964 *A motel operator refused to serve an African American customer *The Supreme Court upheld the Civil Rights Act of 1964, which outlawed discrimination in schools, places of work, voting sites, public accommodations, and public areas

Brandenburg v. Ohio

1969--Determined that a law that proscribes advocacy of violence for political reform is constitutional if applied to speech that is not directed toward producing imminent lawlessness and is not likely to produce such action is not constitutional.

INS v. Chadha

1983, the Supreme Court case that ruled legislative vetoes were unconstitutional, but Congress continues to enact laws containing them.

Massachusetts v. EPA

2007, The EPA must regulate CO2 emissions in accordance with the Clean Air Act.

Amendment II

A well regulated militia, being necessary to the security of a free state, the right of the people to keep and bear arms, shall not be infringed. -- In the 2008 case District of Columbia v. Heller, the Supreme Court held that the "Second Amendment protects an individual right to possess a firearm unconnected with service in a militia, and to use that arm for traditionally lawful purposes, such as self-defense within the home."

In response to the Second World War, Congress passed the Non-Detention Act, which states "no citizen shall be imprisoned or otherwise detained by the United States except pursuant to an Act of Congress." Harry, a U.S. citizen, was captured in Afghanistan and detained by the executive branch as an "enemy combatant." Does the President have the authority to detain Harry?

No. According to Justice Jackson's test, the President's authority is "at its lowest ebb" when he acts in contradiction of the express or implied will of Congress. Here the Congress has prohibited the detention of citizens without congressional authorization, and thus, the President's detainment of Harry without a congressional authorization directly contradicts Congress' Act.

Hester Prynne sues the State of Scarletter in federal court for non-payment of state welfare funds. Will the court hear her case?

No. Her suit will be prohibited by the Eleventh Amendment, which forbids damages actions in federal courts against states, based on past conduct, where the damages will be payable from state funds, absent state consent to the suit or express congressional authorization for the lawsuit. Edelman v. Jordan, 415 U.S. 651 (1974).

Slaughterhouse Cases

A series of post-Civil War Supreme Court cases containing the first judicial pronouncements on the 13th, 14th, and 15th Amendments. The Court held that these amendments had been adopted solely to protect the rights of freed blacks, and could not be extended to guarantee the civil rights of other citizens against deprivations of due process by state governments. These rulings were disapproved by later decisions. -- A 1873 U.S. Supreme Court decision, 83 US 36, on a series of cases in which the Court expressed its first interpretation of the privileges and immunities clause of the Fourteenth Amendment. The court interpreted this clause as protecting the rights people have by virtue of their US citizenship, not by virtue of their citizenship of a state. It then defined the rights of US citizens narrowly, excluding civil rights. The dissents, and modern critics of the case, say that this effectively rendered the clause meaningless.

Dormant Commerce Clause

A situation in which the federal government has the Commerce Clause power to regulate an area of commerce but has chosen not to regulate that area of commerce.

In the Town of Smithville, all homeowners have signed a restrictive covenant that prohibits the sale of a home to non-whites. Ima Bigot, a resident, sues Lincoln, another resident, for damages for selling to a non-white person in violation of the covenant. What result?

According to Shelley v. Kraemer, 334 U.S. 1 (1948), the enforcement of such a covenant would constitute state action violative of the Fourteenth Amendment. (Specifically, it's an equal protection violation.) For the action of a private entity to constitute state action, the state must either have (1) provided a "mantle of authority" for the conduct, or (2) been significantly involved in, or encouraged, the conduct. One common way for this to happen is for the government to enact a non—race-neutral rule of law that facilitates the private discrimination. Granting enforcement of racially restrictive covenants falls into this forbidden category. As a result, Lincoln will prevail.

What race-conscious measures does Justice Kennedy's concurring opinion in the Parents Involved cases permit for public school districts that seek to avoid schools segregated by race?

According to Justice Kennedy, a school district may use the following race-conscious methods: (1) Strategic site selection of new schools, based on awareness of neighborhood demographics; (2) Drawing attendance zones with awareness of neighborhood demographics; (3) Allocating resources for special programs; (4) Recruiting students and faculty in a targeted fashion; and (5) Tracking enrollments, performance, and other statistics by race. MNEMONIC: DARTS (Drawing attendance zones; Allocating resources; Recruiting students; Tracking enrollments; strategic Site selection). N.B.: According to Justice Kennedy, the government may not "treat each student in different fashion solely on the basis of a systematic, individual typing by race." Parents Involved in Community Schools v. Seattle School District, 127 S. Ct. 2738 (2007).

Congress enacts a national marriage tax of $50 that must be paid whenever a couple is married. What enumerated power, if any, authorizes Congress to pass this tax?

According to Article I, §8, cl. 1, "The Congress shall have Power To lay and collect Taxes, Duties, Imposts and Excises." The taxing power is a separate source of power from Congress' Spending and Commerce powers.

Justice Robert Jackson's well-known concurrence in the Steel Seizure case identified three categories of presidential authority. What are the three categories?

According to Justice Jackson: (1) The President is at the height of his authority when he acts with the express or implicit authorization of Congress; (2) The President operates in a "zone of twilight" when he acts in the absence of either a congressional grant or denial of power; and (3) The President's authority is "at its lowest ebb" when he acts in contradiction of the express or implied will of Congress.

Why did the writ of habeas corpus apply in Guantanamo Bay, which is not part of the United States?

According to Justice Kennedy's opinion in Boumediene, the base at Guantanamo Bay was "within the constant jurisdiction of the United States," which "maintains de facto sovereignty over this territory." Boumediene v. Bush, 128 S. Ct. 2229 (2008).

The State of Serenity enacts a statute that requires potato farmers to remove federal labels on potato bags and replace them with state-approved labels, which involves an additional expense to the farmer. One out-of-state grower, Farmer Johannes, challenges this as an invalid state regulation of interstate commerce. What result?

According to the Supremacy Clause, federal law will control, because the state law here directly conflicts with the federal law. McDermott v. Wisconsin, 228 U.S. 115 (1913). There is a direct conflict here, because one can't comply with both the federal and state regulations. Actual conflict cases are the clearest application of the Supremacy Clause. RELATED ISSUE: Say the state regulation required potato farmers to add a state label without replacing the federal label. Then there would be no direct conflict with federal law, because a farmer could comply with both federal and state requirements. Instead, the conflict would be indirect, and the validity of the state requirement would depend on whether Congress intended to preempt the entire potato-labeling field (and, beyond that, whether it unduly burdened interstate commerce, which it almost certainly wouldn't). NOTE: The area would be subject to federal regulation under the Commerce Clause, and subject to state regulation by virtue of state police powers.

What was the Court-packing plan?

After the Court rejected his New Deal legislation, President Franklin Roosevelt proposed to appoint an additional judge for every Supreme Court justice over age 70 who had served at least ten years on the Court. There was great political uproar over the proposal and it was never enacted. On its own, however, the Court changed its limited reading of the Commerce Clause, ushering in a period of broad Commerce Clause power that lasted until 1995.

Sugarman v. Dougall

Alienage Issue: Constitutionality of NY statute that exclude aliens from all government civil service positions filled by competitive examination Rationale: Citizen v. legal aliens = strict scrutiny; citizen/legal alien v. illegal aliens = rational basis Holding: Couldn't discriminate against aliens for lower level civil service jobs, but you could for higher level

Perry v. Sindermann

Allowed public teacher with BREACH of CONTRACT to take case to State or Federal; coach was promised job

Under what circumstances may Congress validly levy a tax?

Almost all circumstances, unless the tax is really a "disguised regulation" (i.e., one that achieves a regulatory effect that could not have been directly achieved by Congress). If a tax is found to be a disguised regulation, it will be struck down unless it falls under some other enumerated power. As long as the tax actually raises substantial revenue or Congress intended that it raise revenue, it won't be considered to be a disguised regulation. NOTE: The "tax versus disguised regulation" issue isn't too important, because almost any tax statute could be sustained as a necessary and proper means of implementing the commerce power. NOTE: The following taxes are also probably valid: (1) a tax with regulatory provisions that bear a reasonable relation to its enforcement; and (2) a tax that regulates directly through its rate structure

Youngstown Sheet & Tube Co. v. Sawyer

Also commonly referred to as The Steel Seizure Case, was a United States Supreme Court decision that limited the power of the President of the United States to seize private property in the absence of either specifically enumerated authority under Article Two of the United States Constitution or statutory authority conferred on him by Congress. It was a "stinging rebuff" to President Harry Truman.

U.S. v. E.C. Knight

Also known as the "'Sugar Trust Case,'" this Supreme Court case that limited the government's power to control monopolies. In 1892 the American Sugar Refining Company gained control of a 98% monopoly of the American sugar refining industry. President Grover Cleveland directed the national government to sue the Company under the provisions of the Sherman Antitrust Act. The court ruled that manufacturing was a local activity not subject to congressional regulation of interstate commerce.

To be considered "interstate commerce," must an interstate activity be "commercial"?

Although the Court has permitted regulation of wheat raised and consumed on the farm where it was raised as interstate commerce (Wickard v. Filburn, 317 U.S. 111 (1942), in two recent cases, the Supreme Court has given significant weight to the fact that the activity Congress tried to regulate was not in itself a commercial activity. In U.S. v. Lopez, 514 U.S. 549 (1995) (involving congressional attempts to combat violence near schools) and in U.S. v. Morrison (2000) (congressional efforts to combat violence against women), the fact that Congress was regulating what the majority found to be "non-economic activity" contributed to the Court's conclusion that the legislation exceeded the scope of the commerce power. So now, it seems that the Court will scrutinize the appropriateness of Congress' reliance on its commerce powers somewhat more closely than where the regulated activity is itself truly "commercial." Nonetheless, in Gonzalez v. Raich, 545 U.S. 1 (2005), the Court upheld Congress' criminalization of intrastate possession of marijuana for medicinal purposes, signaling that Congress may regulate non-commercial activity as part of its broader regulation of commercial activity where Congress reasonably believes that the failure to regulate the non-commercial activity would jeopardize the overall regulatory scheme. Thus, the trend of whether the Court is narrowing or expanding Congress' commerce power is not clear cut.

The State of Sillycon enacts a statute regulating the marketing of high-tech instrumentation. Subsequently, Congress enacts a similar regulation wherein Congress provides that it intends the regulation to cover the whole field of high-tech instrumentation. Will the non-conflicting state law still be valid?

No. If Congress provides for regulation intended to cover the whole area in question, and the area is one in which both Congress and the states have power to regulate (e.g., commerce, with federal power coming from the Commerce Clause and state power from police powers), then the federal law supersedes the state law, regardless of whether the state law directly conflicts with the federal law.

Is interstate commerce immune from state taxation?

No; although Congress can forbid states from taxing interstate commerce, if there is no contrary federal legislation, the Court will balance the burden on commerce against the state's need for revenue, and require interstate commerce to pay a fair share. EXAMPLE: The State of Fandango imposes a 5% tax on interstate telecommunications originating or terminating in state, which is charged to an in-state service address. (There is a credit for any taxpayer proving that he paid a tax in another state on the same call triggering the 5% tax.) Such a tax is constitutional. Goldberg v. Sweet, 488 U.S. 252 (1989).

Strict Scrutiny

As a general matter, strict scrutiny requires that the classification in question must be necessary to promote a compelling governmental interest. Classifications subject to strict scrutiny are called "suspect classes." There are three classifications that are, in general, subject to strict scrutiny—those based on race, those based on alienage, and those that unduly burden, discriminate, or penalize the exercise of a fundamental right. It requires that, for a law to be valid, the law must be necessary to promote a compelling government interest. A statute is only necessary if there are no less discriminatory alternatives available to attain the goal. In the equal protection context, this test applies to state actions that create either a suspect classification and discriminate against a suspect "class," or a classification that unduly burdens, discriminates against, or penalizes the exercise of a fundamental right. NOTE: It is very difficult for a law to pass the strict scrutiny test. NOTE: Explicitly guaranteed rights (like those in the Bill of Rights) are usually analyzed by themselves, not as an equal protection issue (e.g., free speech, freedom of religion). NOTE: The strict scrutiny test is also used for substantive due process problems involving a governmental action that impinges on a fundamental right.

Ingraham v. Wright

U.S. Supreme Court decision stating that corporal punishment does not violate the cruel and unusual punishment clause of the Eighth Amendment.

Griswold v. Connecticut

Established that there is an implied right to privacy in the U.S. Constitution Married couple wanted to get contraceptives; struck down a Connecticut law prohibiting the sale of contraceptives; established the right of privacy through the 4th and 9th amendment

What does the Commerce Clause provide?

Basically, it gives Congress the right to regulate interstate and international commerce. Article I, §8, cl. 3 grants Congress the power "to regulate Commerce with foreign Nations, and among the several states . . ." RATIONALE: To eliminate "economic balkanization" and trade barriers among the states, so that a truly national economy is maintained. NOTE: The Commerce Clause acts both as (1) a source of congressional authority, and (2) as an implicit limitation on state power. NOTE: Article I, §8 also gives Congress power to regulate commerce among American Indian tribes.

Olive Oyl is elected President of the United States. She nominates Bluto to a federal judgeship, and he is approved by the Senate. Once he becomes a judge, it's clear that he's a real Neanderthal when it comes to civil rights. This disappoints Olive Oyl, so she calls him and tells him he's fired. He challenges his dismissal. What result?

Bluto will prevail, because Olive, as President, lacks the power to remove federal judges. Instead, judges are entitled to lifetime tenure "during good behavior" under Article III of the Constitution. RELATED ISSUE: Say that instead of being appointed as a judge, Bluto was made an FTC commissioner. Olive similarly couldn't remove him from office at will, because the President may not remove executive officers with quasi-legislative or quasi-judicial functions. Humphrey's Executor v. U.S., 295 U.S. 602 (1935).

What constitutional provision would justify an act of Congress not explicitly enumerated in the Constitution?

Article I, §8, cl. 18: Congress may "make all Laws which shall be necessary and proper for carrying into Execution the foregoing Powers, and all other Powers vested by this Constitution in the Government of the United States." This "Necessary and Proper" Clause allows Congress to take any legislative action that is rationally related to the carrying out of an objective that falls within an enumerated power. McCulloch v. Maryland, 17 U.S. 316 (1819). NOTE: In addition, the federal government has certain inherent powers, neither enumerated nor implied by the Constitution, which instead are inherent in the concept of national government. The principal inherent power concerns foreign affairs.

To cut the federal deficit, Congress decides to levy a tax on grain exported to other countries. On what basis could this tax be challenged?

Article I, §9 forbids Congress from taxing exports. That's all there is to it.

Zivotofsky v. Kerry

Article II of the Constitution grants the U.S. president the exclusive authority to formally recognize a foreign sovereign through executive power that Congress may not contradict via statute.

What is the source of the federal judicial power?

Article III, §1: The federal judicial power "shall be vested in one Supreme Court, and in such inferior Courts as the Congress may from time to time ordain and establish.".

Hawaii v. Midkiff

CONTRACTS CLAUSE/TAKINGS CLAUSE/REGULATORY TAKINGS

Home Building v. Blaisdell

CONTRACTS CLAUSE/TAKINGS CLAUSE/REGULATORY TAKINGS

Keystone v DeBenedictis

CONTRACTS CLAUSE/TAKINGS CLAUSE/REGULATORY TAKINGS

Penn Central Transportation Co. v. City of New York

CONTRACTS CLAUSE/TAKINGS CLAUSE/REGULATORY TAKINGS

Congress passed the Military Commissions Act (MCA), eliminating the jurisdiction of federal courts to hear habeas petitions from detainees at Guantanamo Bay in Cuba. The government claims that noncitizens detained as enemy combatants in territory outside U.S. borders have no constitutional rights or privilege to habeas corpus. Boum, an Algerian national who has been declared an enemy combatant and held at Guantanamo Bay, challenged his denial of habeas. Who should win?

Boum. The Court in Boumediene v. Bush, 128 S. Ct. 2229 (2008), held by a 5-4 vote that even foreign nationals detained in Guantanamo Bay have a constitutional right to habeas corpus review. The Suspension Clause of Article I, §9, cl. 2 has full effect at Guantanamo. Justice Kennedy's opinion concluded that the jurisdiction-stripping of the Military Commissions Act (MCA) violated the Suspension Clause. Boumediene v. Bush, 128 S. Ct. 2229 (2008). NOTE: Congress must act in accordance with the formal requirements of the Suspension Clause to deny the writ, which the MCA in Boumediene failed to do. NOTE: Chief Justice Roberts' dissent argued that Congress had provided an adequate substitute for habeas with the tribunals of the MCA and therefore had not suspended habeas corpus in violation of the Suspension Clause.

Masses Publishing Co. v. Patten

CONTENT-BASED RESTRICTIONS: DANGEROUS IDEAS AND INFORMATION

Shaffer v. United States

CONTENT-BASED RESTRICTIONS: DANGEROUS IDEAS AND INFORMATION

In 1995, the Supreme Court for the first time in 60 years invalidated a statute as beyond the Commerce Clause power. U.S. v. Lopez, 514 U.S. 549 (1995). This decision indicates that the Supreme Court may be getting back into the business of reviewing congressional actions based on the Commerce Clause. Therefore, it's important to know the three broad categories of activities that Congress can constitutionally regulate, based on the modern view of the commerce power in light of Lopez. What are they?

Channels: Congress can validly regulate use of the "channels" of interstate commerce (e.g., highways and waterways) even though a particular activity may occur wholly intrastate. Instrumentalities: Congress can regulate the instrumentalities used in interstate commerce, even though the regulation affects a solely intrastate activity (e.g., Congress could mandate that every truck used in interstate commerce have a specific safety device, even though a particular truck is used only intrastate). This category refers to people, machines, and other "things" used in carrying out commerce. "Substantially Affecting" Commerce: The biggest category of activities that can be regulated contains those activities that have a "substantial effect" on interstate commerce. The rule governing congressional regulation based on this category depends on whether the activity in question is commercial or non-commercial. Activity is commercial: If the activity itself is arguably commercial, then the "cumulative effect" theory of Wickard v. Filburn, 317 U.S. 111 (1942), applies; Congress can regulate an activity that as a class has a substantial effect on interstate commerce, even if the activity, taken alone, would not. Activity is non-commercial: Under U.S. v. Lopez, 514 U.S. 549 (1995), it now appears that if the activity is not commercial, then Congress can regulate only if there is a pretty obvious connection between the activity and interstate commerce. (Thus in Lopez, the relationship of guns in schools and commerce was found to be too weak a connection to satisfy this test. And in the later case of U.S. v. Morrison, 529 U.S. 598 (2000), the relationship between violence against women and interstate commerce was also too weak—for instance, the fact that some women fail to travel interstate or to work for interstate businesses because they fear violence was found to be too weak a connection to interstate commerce to qualify.) NOTE: It also now appears that the Supreme Court won't give much deference to the fact that Congress "believed" that the activity had the requisite "substantial effect" on interstate commerce.

Due Process Clause

Clause in the 5th and 14th Amendment limiting the power of the national government; states that no one shall be "deprived of life, liberty or property without due process of law."

What's the most common application of the Supremacy Clause?

Commerce Clause problems. In fact, the only time you use a commerce-clause analysis to decide the constitutionality of a state statute impacting interstate commerce is when there's no relevant federal legislation. If there is relevant federal legislation, then you analyze the problem under the Supremacy Clause. You ask three questions: Did Congress expressly authorize or prohibit state regulation (or does the Constitution expressly bar state action— e.g., coining money, conducting foreign affairs)? If so, that authorization or prohibition controls. If not, go on to #2. Is there a direct conflict between the federal and state regulation (e.g., joint compliance isn't possible, or the objectives conflict)? If so, federal law automatically preempts. If there's no direct conflict (i.e., the federal and state statutes merely cover the same subject matter), go on to #3. Was the federal law intended to occupy the entire field? If so, the federal law preempts the state law. If not, the state law stands. NOTE: Supreme Court cases indicate that Congress will be deemed to have preempted an area only where its intent is unmistakable, or where the nature of the regulated subject matter does not permit any other conclusion. Florida Lime & Avocado Growers v. Paul, 373 U.S. 132 (1963).

Gibbons v. Ogden

Commerce clause case (1824). Decision greatly enlarged Congress' interstate commerce clause power by broadly defining the meaning of "commerce" to include virtually all types of economic activity. Riverboat case Pair with Lopez & Morrison cases (limiting commerce power).

Congress enacts a statute that apparently regulates a purely intrastate activity. How can this be a valid exercise of the Commerce Clause?

Congress can regulate purely intrastate activities when (1) the activities are "part of an economic class of activities" and (2) the activities have a substantial effect on interstate commerce. When considering whether the activities have a substantial effect on interstate commerce, the aggregate effect of the activities on interstate commerce can be taken into consideration. Gonzales v. Raich, 545 U.S. 1 (2005).

Taxing Power

Congress has the power to tax, and most taxes will be upheld if they bear some reasonable relationship to revenue production or if Congress has the power to regulate the activity taxed.

What limits do the Tenth Amendment and federalism place on Congress' Commerce Clause powers?

Congress may not use its Commerce Clause powers to "commandeer" the states' legislative or executive powers by forcing the states to enact or administer a federal regulatory program.

Amendment I

Congress shall make no law respecting an establishment of religion, or prohibiting the free exercise thereof; or abridging the freedom of speech, or of the press; or the right of the people peaceably to assemble, and to petition the government for a redress of grievances.

McDonald v. City of Chicago

Constitutional Issue: 2nd Amendment Precedent: States cannot create laws that ban ownership of guns.

What factors determine the procedural safeguards necessary in depriving someone of a liberty or property interest?

Courts employ a balancing test—the importance of the property or liberty interest in question and the risk an erroneous deprivation in a particular procedure would create (considering the probable value of any additional safeguards) are weighed against the importance to the government of the function in question and the administrative and fiscal expense of a particular safeguard. Mathews v. Eldridge, 424 U.S. 319 (1976).

Katzenbach v. Morgan

Upheld federal legislation outlawing state requirements that a prospective voter must demonstrate literacy in English

Procedural due process is required when one is deprived of life, liberty, or property. Is "property" confined to land or chattels?

No; it also covers an interest already acquired in specific benefits. For instance, a high school student has a property interest in his education; welfare recipients have a property interest in continued benefits; drivers have a property interest in retaining their driver's licenses. N.B.: A person's mere "expectation" of maintaining benefits (or a job) isn't sufficient to create a property right; the person must have a "legitimate claim of entitlement" to continued benefits, and this in turn requires a government-created expectancy pursuant to applicable law (e.g., tenure for public school teachers).

Can you remember the three elements of constitutional standing?

MNEMONIC: I Can Remember: INJURY CAUSATION REDRESSABILITY

Ferguson v. Skrupa

Debt adjusting OK. As long as state law doesn't violate federal law or constitutional problem, it is valid.

Hammer v. Dagenhart

Declared the Keating-Owen Act (a child labor act) unconstitutional on the grounds that it was an invasion of state authority.

Veto-gate. President Pauli has promised Ed Executive that she will veto a regulatory bill that has just passed Congress limiting the pay of publicly traded corporations' executives. The bill was born of the public—and Congress's—dismay at the large disparities between what the top earners made at various publicly traded corporations and what those at the bottom took home. To his dismay, however, the President soon reverses course and signs the bill, later claiming that she was persuaded that the curb was, indeed, needed. Furious because his own pay is due to be decimated, Ed files suit in federal district court seeking judicial review of the President's veto. A reviewing court is likely to

Deny review because the decision whether to veto a bill or not is within the President's discretion.

State statutes involving legal aliens are subject to strict scrutiny (except for "right to govern" situations). What about illegal aliens?

Discrimination against illegal aliens is not subject to strict scrutiny. However, where there's no congressional policy favoring a state rule discriminating against illegal aliens, the state rule must meet "intermediate" scrutiny—that is, it must be substantially related to the achievement of an important state goal. (Note that a state could give preferential treatment to citizens and legal resident aliens in allocating government benefits.) Plyler v. Doe, 457 U.S. 202 (1982). EXAMPLE: A state cannot deny a free, public education to the children of illegal aliens, because: The children have no control over their status; and The burden of a lack of education is enormous and lasting. NOTE: There haven't been significant cases on adult illegal aliens other than Plyler v. Doe; it's not clear how significant it was that the class of people at issue were not those who had intentionally decided to illegally enter the country, but rather their children, who had done nothing wrong.

U.S. Department of Agriculture v. Moreno

EQUAL PROTECTION METHODOLOGY: RATIONAL BASIS REVIEW

San Antonio Independent School District v. Rodriguez

Education

Plyler v. Doe

Education states can't deny education for illegal immigrants amendment 14

Under what circumstances may a state tax activities involving interstate commerce?

First, the state may tax such commerce if Congress expressly so consents. When there is no guidance from Congress, the validity of the tax will depend on whether it discriminates against interstate commerce. If it is discriminatory (e.g., it singles out interstate commerce for taxation, and there's no similar tax on local commerce), it virtually automatically violates the Commerce Clause. (It would also violate the interstate Privileges & Immunities Clause, Art. IV, §2 and perhaps, the Equal Protection Clause if the discrimination isn't rationally related to a legitimate state purpose.) Ch.6-II(C). If it's non-discriminatory, the court uses a balancing test to determine the validity of the tax on interstate commerce. The court balances the state's need for revenue versus the burden on the free flow of commerce. The burden generally will be too great under the Commerce Clause, if multiple burdens are imposed on an interstate activity by various state taxes. It will be invalid under the Due Process Clause if there are "insufficient contacts" between the subject matter and the state to justify the state's taxing the subject matter (because the subject matter would receive insufficient benefits from the state to justify the tax).

Takings Clause

Government can take private property for a public purpose, but it must provide fair compensation to the owners of that property.

What is a writ of habeas corpus?

Habeas corpus is Latin for "you have the body." A writ of habeas corpus is used by a prisoner to challenge the government's imprisonment of the prisoner's body. The writ forces the government into federal court to explain or justify why the prisoner is being held. If the judge decides the prisoner is being held in violation of the Constitution or federal law, the judge may order release so that the government no longer has the body.

US v. Carolene Products

Hughes court, milk with additives banned from interstate shipment, upheld federal policy, sets ground work for the assumption of constitutionality in economic regulation in stones footnote

Whenever Congress regulates the states, you must consider whether the Tenth Amendment applies.

If Congress "commandeers" the state legislative or executive branch to do its work, the Tenth Amendment is violated. Under Printz, e.g., state sheriffs can't be forced to keep federal gun ownership records. Congress may pass laws that regulate states and private actors in the same way without violating the Tenth Amendment. Congress may regulate the states when they participate in the market. If both states and private companies sell personal information to database owners, both may be regulated without violating the Tenth Amendment.

Congress passes the Child Labor Act, which prohibits the transportation in interstate commerce of any articles produced by companies employing children under 12 years old. Is the statute constitutional?

In 1918, no. In 1941, yes. The Court in Hammer v. Dagenhart, 247 U.S. 251 (1918), invalidated a similar statute on the grounds that the employment was not directly related to interstate commerce even though the articles traveled in interstate commerce. Hammer was overruled by U.S. v. Darby, 312 U.S. 100 (1941). The Court has identified different theories of the Commerce Clause at different stages of its history.

Whole Women's Health v. Hellerstedt

In 2013, the Texas Legislature passed House Bill 2 (H.B. 2), which contained several provisions related to abortions. One such provision required that any physician performing an abortion have admitting privileges at a hospital within 30 miles of where the abortion was performed, and another provision required that all abortion clinics comply with standards for ambulatory surgical centers. The petitioners are a group of abortion providers who sued the State of Texas seeking to invalidate those provisions in H.B. 2 as they relate to facilities in McAllen and El Paso. The petitioners argued that H.B. 2 denied equal protection, unlawfully delegated lawmaking authority, and constituted arbitrary and unreasonable state action. The district court dismissed the equal protection, unlawful delegation, and arbitrary and unreasonable state action claims and granted declaratory and injunctive relief against the enforcement of the two contested provisions of H.B. 2. The U.S. Court of Appeals for the Fifth Circuit affirmed the district court's dismissal of the equal protection, unlawful delegation, and arbitrary and unreasonable state action claims and partially reversed the injunctions because the plaintiffs failed to show that they placed a substantial burden in the path of a woman seeking an abortion.

Amendment VI

In all criminal prosecutions, the accused shall enjoy the right to a speedy and public trial, by an impartial jury of the state and district wherein the crime shall have been committed, which district shall have been previously ascertained by law, and to be informed of the nature and cause of the accusation; to be confronted with the witnesses against him; to have compulsory process for obtaining witnesses in his favor, and to have the assistance of counsel for his defense. -- The Sixth Amendment guarantees the rights of criminal defendants, including the right to a public trial without unnecessary delay, the right to a lawyer, the right to an impartial jury, and the right to know who your accusers are and the nature of the charges and evidence against you. It has been most visibly tested in a series of cases involving terrorism, but much more often figures in cases that involve (for example) jury selection or the protection of witnesses, including victims of sex crimes as well as witnesses in need of protection from retaliation.

Amendment VII

In suits at common law, where the value in controversy shall exceed twenty dollars, the right of trial by jury shall be preserved, and no fact tried by a jury, shall be otherwise reexamined in any court of the United States, than according to the rules of the common law. -- The Seventh Amendment continues a practice from English common law of distinguishing civil claims which must be tried before a jury (absent waiver by the parties) from claims and issues that may be heard by a judge alone. It only governs federal civil courts and has no application to civil courts set up by the states when those courts are hearing only disputes of state law.

When does state involvement in private discrimination rise to the level of "state action" for constitutional purposes?

In the typical state action case, it's when either: (1) the state has somehow provided a mantle of authority for the private conduct; or (2) the state was significantly involved in, or encouraged, the private conduct. There are three typical ways this can happen: (1) The state creates a legal framework governing the conduct (e.g., a court enforces a racially restrictive covenant; a statute requires segregated public toilets). Note that the involvement must be non-neutral (e.g., enforcing a neutral trespass law when a minority member is excluded from private property is not state action); (2) The state delegates authority to a private actor (the function must be one traditionally associated with governments and historically operated exclusively by government entities, including running elections and governing towns and cities ["company towns"], but not including public utilities or shopping centers); or (3) The state knowingly accepts benefits derived from unconstitutional behavior (e.g., state-run parking garage gets vital rents from privately owned restaurant that discriminates). VIOLATIONS COVERED: Anything other than the Thirteenth Amendment (which already addresses private conduct involving slavery), including due process, equal protection, and First Amendment rights.

Intermediate Scrutiny

Intermediate scrutiny requires that the classification in question be substantially related to an important governmental interest. Classifications subject to intermediate scrutiny are called "quasi-suspect." There are two classifications that qualify for this treatment—gender and illegitimacy. (Gender classifications must be supported by an "exceedingly persuasive justification.")

ANALYZING STATE STATUTES AFFECTING INTERSTATE COMMERCE

Is there relevant federal legislation? If so, there's a preemption issue, not a Commerce Clause issue. If not, go on to #2. Does the statute discriminate against interstate commerce—that is, does it either intentionally or unintentionally favor local economic interests over the economic interests of other states? If it does, the statute is invalid without Congress' consent to so discriminate, regardless of whether the burden would otherwise be permissible, unless either (a) it is designed to protect health and safety interests, it is reasonable, and no non-discriminatory alternatives are available; or (b) the state is a "market participant," in which case it can discriminate in favor of local businesses as long as it doesn't violate the interstate Privileges & Immunities Clause of Article IV (watch for this any time a state or city interferes with private sector employment). If the statute is non-discriminatory in both intent and effect, a balancing test is used to determine whether the burden on interstate commerce is unreasonable, thus invalidating the regulation: the burden on interstate commerce (e.g., cost and difficulty of compliance, inefficiency created, existence of less burdensome alternatives) versus the strength of the state interest in the regulation. N.B.: There is a strong presumption of validity of non-discriminatory statutes where the state interest is health, safety, or social welfare; on the other hand, a court is not likely to uphold the statute where the state interest is economic (i.e., aiding local business interests). NOTE: Non-discriminatory regulation enjoys a presumption of constitutionality that can only be overcome by a clear showing that the state benefit is outweighed by a national interest in uniformity or the free flow of commerce. NOTE: Congress can't authorize state violations of constitutional provisions other than the Commerce Clause (e.g., Due Process and Equal Protection).

New York v. United States

Issue: Does the Low-Level Waste Act violate the 10th Amendment and "guarantee clause" of Article 4? Holding: Congress has substantial power under Constitution to encourage, but not compel, the states to provide for the disposal of the radioactive waste generated within their borders Rationale: In looking 10th Amendment, Court is required to determine whether an incident of state sovereignty is protected by a limitation on an Article I power. If encouraged but not coerced, state governments remain responsive and accountable to their people. But if coerced not encouraged, state governments do not have to remain responsive and accountable to their people.

Board of Trustees v. Garrett

Issue: May someone sue a state for damages in federal court under Americans w/Disabilities Act?? Holding: No. Suits in fed. court by state employees to recover $ by reason of state's failure to comply with ADA are barred by the 11th Amendment Concurrence: Alleged violations could not have been so serious b/c not a lot of people who sued for it Dissent: There was contrary Congressional finding in; rational basis refers to the judgment of Congress

What are political questions?

Issues that are considered "non-justiciable," because they are: Issues the Constitution commits to another governmental branch (based on separation-of-powers principles); or Issues the judicial process is inherently incapable of resolving and enforcing because there are no standards for resolving the dispute. EXAMPLE: Foreign relations. CRITERIA FOR DETERMINING WHETHER A CASE PRESENTS A POLITICAL QUESTION (at least one must be present): A "textually demonstrable" constitutional commitment of the issue to Congress or the executive branch; Lack of "judicially discoverable and manageable standards"; Need for a single pronouncement from the branches; Difficulty or impossibility of devising effective judicial remedies. Baker v. Carr, 369 U.S. 186 (1962). N.B.: "Political question" is a misnomer, in that some issues involving political rights are clearly justiciable (e.g., deprivation of the right to vote in general elections).

President Rozsman appoints Professor Nyarady to head the Federal Resources Board. President Rozsman obtains the advice and consent of the Senate for the appointment. Can President Rozsman remove Professor Nyarady without the approval of the Senate?

It depends on the status of the Board. If the Board is considered solely an arm of the executive branch, appointees are removable by the President at will—even if the original appointment required the Senate's advice and consent. But if the Board is considered quasi-legislative or quasi-judicial (e.g., the FTC), the President doesn't have the power to remove its members at will. RATIONALE: Congressional control over removal of quasi-legislative and quasi-judicial officials is necessary to preserve their independence from the executive branch. NOTE: For an administrative agency like the FTC, which was created by an act of Congress that lists the term of office, removal may be limited to specific causes enumerated in the statute creating the agency.

How is the "removal power" split between Congress and the President?

It depends on who is being "removed." (1) Purely executive officers appointed by the President (e.g., cabinet members)—the President has the exclusive power to remove if the appointee is a principal officer, such as a Cabinet member. However, if the appointee is an inferior officer (e.g., special prosecutor), Congress may limit the President's removal power if such limitations do not interfere with the President's ability to perform his constitutional duties. (2) Congress vests appointment power in some official other than the President—Congress may regulate and limit removal. (3) Officer is quasi-legislative or quasi-judicial (e.g., FTC commissioner)—normally, the President holds the removal power. However, Congress may limit or completely block the President's right of removal for these officers. (4) Executive department in question has administrative regulations governing employee removal—the department must follow those regulations (although it can always change the regulations).

In the Constitutional Law context, what does "incorporation" mean?

It means that a provision of the Bill of Rights is applicable to the states through operation of the Fourteenth Amendment's Due Process Clause. Originally, the Bill of Rights was only applicable to the federal government; however, over time most of the provisions have been "incorporated" so as to bind the states as well.

What does "ripeness" mean?

It means that an issue presents an immediate threat of harm, rather than the injury being speculative and possibly never occurring. An issue must be "ripe" for a court to resolve it.

What does the Supremacy Clause do?

It provides that any state or local law conflicting with a valid federal law must be struck down. NOTE: The Supremacy Clause does not provide a source of power for the federal government!

What powers does the President enjoy as Commander in Chief of the armed forces?

It's not entirely clear, but such powers do include: Committing armed forces to repel a sudden attack on the United States itself (or any internal insurrection), even before Congress declares war; Legislative power in "theaters of war" (e.g., establishing military governments in occupied territories); and Controlling the disposition (placement) of armed forces. NOTE: The President does not have the power to declare war; that power belongs to Congress (although, in his role as Commander in Chief of the armed forces, the President may conduct military activity in actual hostilities against the United States without such a declaration, under Article II, §2).

substantive due process

Judicial interpretation of the Fifth and Fourteenth Amendments' due process clauses that protects citizens from arbitrary or unjust state or federal laws. -Constitutional requirement that governments act reasonably and that the substance of the laws themselves be fair and reasonable; limits what a government may do.

Johnny Appleseed roams from state to state, taking short-term jobs in orchards during his travels. He never stays in a state longer than one year; however, he currently lives in the State of Bighorn, where he's lived for six months, and he plans to stay indefinitely. Bighorn has a one-year wait before a new resident will be entitled to certain state benefits and services. What kinds of benefits and services may validly require this one-year residency period?

Keeping in mind that the right to interstate travel is fundamental, a one-year residency requirement would not pose an invalid burden on one seeking these services: (1) Reduced ("in-state") tuition at state universities and colleges; and (2) Divorce. RELATED ISSUE: One year would be too long a delay to obtain these benefits: (1) Welfare; (2) Medical care; or (3) Voting (50-day residency is close to the upper constitutional limit).

Lala Bamba, an alien from Colombia, has been convicted of murder in a state court. She was not permitted to obtain counsel. When she challenges her conviction on grounds that the lack of counsel violated her due process rights under the federal Constitution, the government claims that aliens aren't protected by the Due Process Clause. Who's correct?

Lala Bamba; aliens (and corporations) are considered "persons" for purposes of due process.

Amendment V

No person shall be held to answer for a capital, or otherwise infamous crime, unless on a presentment or indictment of a grand jury, except in cases arising in the land or naval forces, or in the militia, when in actual service in time of war or public danger; nor shall any person be subject for the same offense to be twice put in jeopardy of life or limb; nor shall be compelled in any criminal case to be a witness against himself, nor be deprived of life, liberty, or property, without due process of law; nor shall private property be taken for public use, without just compensation. -- The Fifth Amendment creates a number of rights relevant to both criminal and civil legal proceedings. In criminal cases, the Fifth Amendment guarantees the right to a grand jury, forbids "double jeopardy," and protects against self-incrimination. It also requires that "due process of law" be part of any proceeding that denies a citizen "life, liberty or property" and requires the government to compensate citizens when it takes private property for public use.

Amendment III

No soldier shall, in time of peace be quartered in any house, without the consent of the owner, nor in time of war, but in a manner to be prescribed by law. -- Described by some as "a preference for the Civilian over the Military," the Third Amendment forbids the forcible housing of military personnel in a citizen's home during peacetime and requires the process to be "prescribed by law" in times of war. This Amendment is not considered controversial and has never been litigated before the United States Supreme Court.

The State of Warner is one of the few with a population of the rare yellow-bellied sapsucker. A state statute forbids hunting and trapping yellow-bellied sapsuckers and authorizes game wardens to seize and destroy any traps they find. A federal consumer safety regulation regulates the design of such traps to prevent them from pinching the fingers of people who set the traps. Does the federal regulation preempt the state statute?

No, because the regulations have different purposes, a fact that indicates that the two regulations can co-exist. When federal and state laws conflict, the federal law takes precedence under the Supremacy Clause. If there's no direct conflict, the state law is only preempted if the federal law was intended to preempt the entire field. Here, there's no conflict at all. Instead, the laws have two very different purposes—the federal law was designed to make the traps safe, and the state law was intended to protect the animals. The state law does not impede operation of the federal law, and there's no indication the federal law was intended to preempt the entire field of yellow-bellied sapsucker traps. As a result, the state law will stand. NOTE: If the objectives behind the two regulations were incompatible, the state regulation would fail, even if the state and federal regulations didn't conflict on their face.

Are the powers of the state governments listed in the Constitution just as Congress' are?

No, the powers of the state governments are not enumerated like the congressional powers are. According to the Tenth Amendment, "[t]he powers not delegated to the United States by the Constitution, nor prohibited by it to the States, are reserved to the States respectively, or to the people." NOTE: The federal government doesn't have general police powers but the states do.

The State of Suffering has a state constitutional provision that requires a local referendum as a prerequisite to the construction of low-rent public housing projects. Bob Cratchit brings an action in federal court claiming that the statute discriminates against the poor. Bob claims that poverty should be a suspect classification so as to require a compelling state interest before the statute may be upheld. Is he correct?

No. "Poverty standing alone is not suspect classification." Harris v. McRae, 448 U.S. 297 (1980). Therefore, the court would employ the traditional rational relation test—the statute need only be rationally related to a legitimate state goal to be valid. The statute would be upheld on this basis.

Was President Truman's seizure of the steel mills authorized by the President's inherent executive authority to enforce the laws?

No. According to Justice Black's opinion for the Court, Congress had not passed legislation authorizing the President's conduct. Therefore the President was making law, not executing the law, and thus acted unconstitutionally.

Members of Congress become suspicious of the ethics of the Secretary of Defense when it comes to light that a defense contractor gave the Secretary's seven-year-old son a radio-controlled nuclear submarine for his birthday. As a result, Congress passes legislation requiring the Attorney General to investigate any allegations of wrongdoing against cabinet members. If she believes further investigation is warranted, the legislation requires her to apply to a special federal court for the appointment of a special prosecutor to pursue the investigation (and any ensuing prosecution). Once appointed, the special prosecutor can be removed only by the Attorney General, and only for "cause" (any condition substantially impairing the performance of his duties). Does this create a constitutional problem of separation of powers?

No. Although enforcing the law is considered a purely executive function, the legislation here doesn't violate that function, because the powers retained by the President or his appointee (e.g., to decide whether to apply for a special prosecutor at all, and to remove the special prosecutor for cause) allow the President to retain sufficient control to carry out his constitutional duties. Morrison v. Olson, 487 U.S. 654 (1988).

Snow White, who is pregnant, seeks to challenge a state statute restricting the right to an abortion in the first trimester. By the time the case reaches the U.S. Supreme Court, Snow White is, surprisingly enough, no longer pregnant. Is the issue moot?

No. An issue is "moot" if the controversy has been resolved. Normally, a court will not decide a case if the issue has been resolved at any phase of review. However, one exception to the rule covers facts like those here—"capable of repetition yet evading review"—that is, where the action is likely to happen repeatedly, but will always avoid review due to the time required to litigate versus the time the action takes. In pregnancy cases, the time it takes to reach the Supreme Court would virtually always exceed the gestation period; thus, in such cases, the case will not be moot. Roe v. Wade, 410 U.S. 113 (1973).

Can Congress give lower federal courts jurisdiction over cases between two state citizens where no federal issues are involved?

No. Congress cannot authorize federal courts to hear cases that are not within the federal judicial power set forth in Article III.

Congress passed legislation imposing limited sanctions on the foreign Nation of Apathy to encourage it to act more democratically. The U.S. State of Indignation is critical of the numerous human rights violations committed by Apathy. State of Indignation therefore passes legislation prohibiting the state and its agencies from buying goods or services from companies that do business with Apathy. Is the state law valid?

No. Even though Congress and the State of Indignation have similar goals, Congress' action preempts the state from passing sanctions. In the area of foreign policy, the Court is likely to find state action preempted by Congress, even where, as here, Congress did not expressly preempt the state legislation and the state and federal goals are consistent. The Court unanimously found preemption on similar facts in Crosby v. National Foreign Trade Council, 530 U.S. 363 (2000).

Because of its concern for interstate waste disposal, which has a substantial effect on interstate commerce, Congress used its commerce power to pass the Low-Level Radioactive Waste Policy Act. One section of the Act, the so-called take title provision, requires any state that does not enact regulations providing for disposal of its waste to "take title" to any waste within its borders and to be liable for damages connected to the waste's disposal. The State of Waste challenges the take title provisions. Are the take title provisions constitutional?

No. Even though Congress has commerce power to regulate interstate waste, the Tenth Amendment does not allow Congress to "commandeer" State of Waste's legislature. Congress may not directly compel the states to enact and enforce a federal regulatory program. The choice given to the State of Waste by Congress—either to enact legislation or to take title to the waste—is impermissible. New York v. U.S., 505 U.S. 144 (1992). NOTE: Congress could use its spending power to encourage the states to undertake programs regulating waste by promising them funding. NOTE: Congress may directly set standards for waste disposal by the states.

Lizzie Borden is a mentally unbalanced ten-year-old. Will a prior adversary hearing be necessary before she can be committed to a mental institution?

No. First, procedural due process rights only attach when one is deprived of a property or liberty right. Deprivation of one's freedom is a liberty right that clearly would qualify for protection. Second, determining which procedural safeguards are called for requires balancing (1) the strength of the property or liberty interest in question, the risk of erroneous deprivation of the interest through applicable procedures, and the probable value of the additional or substitute procedural safeguards, against (2) the government's interest in the function involved, and the fiscal and administrative expense that the additional procedural safeguards would entail. Mathews v. Eldridge, 424 U.S. 319 (1976). As it applies to the involuntary commitment of children, due process mandates that children can be committed without a prior adversary hearing under limited circumstances, under Parham v. J. R., 442 U.S. 584 (1979). Thus, as long as (1) there is a precommitment inquiry to determine if statutory requirements are present, conducted by a neutral fact finder (e.g., a physician); (2) the inquiry includes a close probe into the child's background; and (3) the opportunity for a periodic review of the commitment is provided, no "adversary" hearing (i.e., no hearing at which Lizzie is represented, can present evidence, and can cross-examine adverse witnesses) will be required. RELATED ISSUE: For involuntary commitment of adults, there must be "clear and convincing evidence" to satisfy the sufferer's procedural due process right. Addington v. Texas, 441 U.S. 418 (1979).

Champion v. Ames

Upheld law prohibiting transport of lottery tickets across state lines. Congress can't regulate what happens within states, but can keep harmful products from crossing state lines.

Officials in the State of Segregation argue that they are not bound by the lower federal court order to desegregate the public schools. Are they correct?

No. In Cooper v. Aaron, 358 U.S. 1 (1958), the Supreme Court rejected the argument of Arkansas officials that they did not have to comply with Brown v. Board of Education, 347 U.S. 483 (1954), stating that "the federal judiciary is supreme in the exposition of the law of the Constitution." NOTE: Compare the Supremacy Clause of Article VI of the Constitution, which makes federal law supreme over state law: "This Constitution, and the Laws of the United States which shall be made in Pursuance thereof; and all Treaties made, or which shall be made, under the Authority of the United States, shall be the supreme Law of the Land." NOTE: The text of the Constitution does not say that the federal judiciary is supreme in interpreting the Constitution. The Court said that it was supreme in interpreting the Constitution in Cooper v. Aaron.

Public School District has no history of racial discrimination. The student population is 41% Caucasian and 59% non-Caucasian. The district allows incoming ninth graders to list their preferences for what school they want to attend. Public School District's high schools are oversubscribed, however, so all students cannot receive their top choice. If too many students list the same school as their top choice, a series of "tiebreakers" determines who can attend what school. The first tiebreaker is whether the student has a sibling at that school. The second tiebreaker depends on the racial composition of the school and the race of the student. If an oversubscribed school is not within ten points of the district's Caucasian and non-Caucasian population, the tiebreaker selects students of the race that will reset the racial balance in the school. Because Public School 101 is 52% Caucasian, Wanda White lost the tiebreaker to an African American student who gained admission to Public School 101. Is the tiebreaker constitutional?

No. In Parents Involved in Community Schools v. Seattle School District, 127 S. Ct. 2738 (2007), in the Roberts Court's first consideration of affirmative action, the Court invalidated a similar program in Seattle by a 5-4 vote. In a plurality opinion by Chief Justice Roberts, four members of the Court concluded that any use of race in the context of school admissions is unconstitutional. The plurality found that the school district had no compelling interest for the program. First, there was no history of past discrimination. Second, the plurality wrote that the government's compelling interest is in the broad diversity of the student body, not the racial diversity targeted by Seattle. A concurrence by Justice Kennedy stated that the government may still consider race to achieve diversity and avoid resegregation. Kennedy rejected the Seattle plan because he believed it engaged in "individual typing by race"; instead, Seattle could have conducted a more individualized review of the applications or used race-neutral means to achieve its goals. The dissenters argued that the plurality had misinterpreted Brown and Grutter.

State of Justice passes the Holocaust Victim Insurance Relief Act, which requires any European insurance company doing business in state to disclose whether it issued insurance policies during the Holocaust. If the European insurance companies fail to comply with the law, they will lose their licenses to do business in the State of Justice. There is no federal statute on this topic. Is the Holocaust Act valid?

No. In a 5-4 decision, the Court ruled that a similar state statute interfered with the President's ability to negotiate executive agreements in this area of foreign policy and therefore was preempted. Even though the President had chosen not to pursue a policy in this area, the state was not allowed to enact a contrary policy because any such policy would interfere with the President's choices. American Insurance Ass'n v. Garamendi, 539 U.S. 396 (2003).

State of Warden is concerned that there are too many frivolous lawsuits by prisoners against state prison officials that are clogging the courts. In response to this problem, Warden sets up a special state Court of Claims that will hear civil rights cases against prison officials only. The Court of Claims is a court of limited jurisdiction where prisoners will not be allowed punitive damages or attorneys' fees. Cases against prison officials brought under the federal civil rights statutes will be heard exclusively in the Court of Claims. Is the State Court of Claims constitutional?

No. In a 5-4 vote in Haywood v. Drown, 129 S. Ct. 2108 (2009), Justice Stevens wrote that a similar court system violates the Supremacy Clause, Article VI, cl. 2, because the state's policy is "contrary to Congress' judgment that all persons who violate federal rights while acting under color of state law shall be held liable for damages . . . a State may not . . . relieve congestion in its courts by declaring a whole category of federal claims to be frivolous."

What provision of the Constitution gives the Supreme Court its power to determine the constitutionality of acts performed by the other governmental branches?

None! No constitutional provision explicitly provides for judicial review. The Supreme Court created this power by means of its interpretation of the Constitution. The Court held that it had the power to declare unconstitutional the acts of other branches of the government (Marbury v. Madison, 1 Cranch 137 (1803)), to declare state statutes unconstitutional (Fletcher v. Peck, 10 U.S. 87 (1810)), and to review the judgment of state courts in cases that fall within the federal judicial power (Martin v. Hunter's Lessee, 1 Wheat. 304 (1816)).

Under the federal Family and Medical Leave Act (FMLA), Congress authorized employees to take up to 12 weeks of unpaid family leave to care for family members who suffer from serious health conditions. Congress passed the FMLA because it had extensive evidence of gender discrimination by the states in the administration of employee leave benefits and desired to remedy that discrimination with the requirements of the FMLA. Dibbs, an employee of the State Department of Human Resources, sued the State after it denied his request for family leave. Is the lawsuit barred by the Eleventh Amendment?

No. In passing the FMLA, Congress relied on its §5 of the Fourteenth Amendment power to enforce the Equal Protection Clause. In Nevada Dept. of Human Resources v. Hibbs, 538 U.S. 721 (2003), the Court ruled that Congress abrogated the states' sovereign immunity in the FMLA because the legislation was a congruent and proportional solution to the problem of gender discrimination. N.B.: The Court has also held that a state does not have immunity in a lawsuit, brought under the Americans with Disabilities Act (ADA), where the disabled plaintiff alleged that he was denied access to the courts. Because access to the courts is a fundamental right protected by the Fourteenth Amendment, the Court upheld Congress' §5 power to abrogate state sovereign immunity. In general lawsuits under the ADA, however, where employees claim that they were fired because of their disabilities, Congress lacks the power to abrogate state sovereign immunity.

Congress passed the Clean Water Act (CWA) to protect "the navigable waters of the United States, adjoining shorelines and other natural resources of the United States" against harm from environmental accidents. To protect those waters and resources, the CWA assigns a monetary penalty not to exceed $10,000 per day against any person or corporation that is responsible for an oil spill onto the waters of the United States. Big Oil Company negligently spilled oil into the water surrounding the State of Salmon. The fishermen of the State of Salmon, whose jobs were affected by the spill, and their fellow citizens, whose health was affected by the spill, sue Big Oil for their economic loss and personal injury. Are their lawsuits preempted by the Act?

No. In the real case about the Alaska oil spill, the Court ruled that Congress had not expressed any intent to occupy the field in the Clean Water Act. Nor was there any reason for the Court to conclude that Congress would have any interest in preempting state tort law, which compensates injuries to persons, in an act whose goal was to protect natural resources. Exxon Shipping Co. v. Baker, 128 S. Ct. 2605 (2008).

In response to the terrorist attacks of September 11, 2001, President George W. Bush issued an executive order providing that non-U.S. citizens who were suspected of aiding the war on terrorism would be tried by special military commissions. The president's tribunals offer less protection to defendants than the procedures established by a congressional statute called the Uniform Code of Military Justice. Hamdan, a citizen of Yemen, was arrested in Afghanistan and charged with assisting Al Qaeda with planning the 9/11 attacks. Can Hamdan be tried for war crimes before one of President Bush's military tribunals?

No. Justice Stevens' opinion for the Court in Hamdan v. Rumsfeld, 548 U.S. 557 (2006), concluded that Congress had not authorized the tribunals and that the President did not have independent authority to create them. According to Justice Kennedy's concurrence, because Congress had already passed legislation setting the scope of military tribunals, the President acted in opposition to existing law—i.e., in Category 3 of the Steel Seizure categories, where the President's authority is at its lowest ebb. NOTE: Once Congress authorizes military commissions, the President acts within his powers to enforce them, as long as they do not violate other constitutional provisions.

The State of Bible has established a program in which any private school—parochial or secular—may have teachers from the public schools sent in to do remedial work in English and math. The remedial instruction is given during regular school hours in the private school, but only to students who are in danger of failing. The teachers' salaries are paid by the state. The private school has no influence over the curriculum for the remedial teaching. 95% of the state dollars spent on the program are spent for instruction at parochial schools. The effect of the program for participating parochial schools is that the school has much more money left to spend on religious instruction. Is the state program a violation of the Establishment Clause?

No. On similar facts, the Court held (by 5-4) that the remedial instruction program was constitutional. See Agostini v. Felton, 521 U.S. 203 (1997), which reversed prior law. The Agostini Court rejected claims that such a program would lead to "excessive entanglement" between government and religion—any need to ensure that the public teachers didn't end up teaching religion could be solved by monthly visits by state inspectors, a participation too small to amount to entanglement.

The State of Factoria has a statute requiring that all police officers take an oral test to ensure that they can communicate adequately in English. Although the test appears racially neutral, a disproportionate percentage of African Americans fail it. The test is administered fairly, and the legislature did not intend to discriminate. Will the test be subject to strict scrutiny?

No; de facto discrimination—where there is a disparate effect, but the statute is facially neutral, administered fairly, and the legislature did not intend to discriminate—is not subject to strict scrutiny, even if a "suspect class" is affected. Instead, the rational relation test will be used: The statute will be upheld if it is rationally related to a legitimate state interest. Because it is at least "legitimate" for government to ensure that its police officers are able to communicate in English, the statute will survive a constitutional attack. Washington v. Davis, 426 U.S. 229 (1976).

Obergefell v. Hodges

Upholds same sex marriage via 14th amendment equal protection clause

Suzi Queezi was feeling nauseous when she arrived at the hospital, where her physician, Doctor Donna, prescribed Tummis to take care of the problem. Tummis can be taken orally or administered directly into the vein by the IV-push method. Doctor Donna opted for the latter approach even though it is more dangerous than giving Tummis orally. Doctor Donna mistakenly put the IV-push into an artery instead of a vein. Queezi contracted severe blood poisoning and lost her leg. Queezi sued in state court on a state tort law claim that Tummis should have been labeled to warn doctors that the IV-push method is dangerous. Tummis' label was approved by the Food and Drug Administration (FDA), which is authorized to "make drug labeling decisions that strike a balance between competing objectives." Is Queezi's lawsuit preempted?

No. On similar facts, the Court ruled against preemption in Wyeth v. Levine, 129 S. Ct. 1187 (2009). First, the Court concluded there was no express preemption as in the Riegel case on the prior card. The Court then ruled that there is a presumption against implied preemption in order to protect the states' police powers. The presumption held in Wyeth because Congress knows how to write an express preemption statute and failed to do so in the labeling context. NOTE: It is easier to win on express preemption claims because of the presumption against implied preemption claims.

The State of Military prides itself on its long military tradition. The state has for many years maintained one (and only one) state-sponsored military academy, the Military Military Academy (MMA), and it has always limited that academy to men. The MMA has a program emphasizing extreme physical rigor and extensive hazing, and the state believes that this type of program would not appeal to the vast majority of women. Is the academy's single-sex limitation constitutional?

No. On these facts, the Court in U.S. v. Virginia, 518 U.S. 515 (1996), held that restricting attendance at Virginia Military Institute violated the equal protection rights of women. The Court announced a new standard for gender-based classifications—they must be supported by "exceedingly persuasive justification," and will be subjected to "skeptical scrutiny." The majority said that as long as there are some women who would be interested in, and qualified for, this type of rigorous and hazing-based program, the fact that most women aren't is irrelevant. NOTE: The Court also expressed its skepticism about any gender-based classifications that are based on "overbroad generalizations" or stereotypes about men and women. So watch out for stereotypes; they should not be able to survive intermediate scrutiny. NOTE: As the result of U.S. v. Virginia, one big change is that in gender-based cases, only objectives that are shown to have actually (not just hypothetically) motivated the government may be considered as satisfying the "important governmental objective" requirement.

The President receives a subpoena to produce certain evidence needed by the prosecution in an upcoming criminal trial. The evidence consists of notes taken by the President during meetings with his advisers, concerning various issues of domestic politics. It's to be used by the prosecution in a prosecution of one of those advisers for violating citizens' rights under color of law. Can the President successfully claim that the doctrine of executive privilege prevents him from being compelled to disclose this information?

No. Presidents do indeed possess an executive privilege—that is, a right to maintain the confidentiality of communications to which they become a party during their performance of their official duties. However, this privilege is "qualified," not absolute. It must yield to the need to develop all the facts needed for a criminal trial. U.S. v. Nixon, 418 U.S. 683 (1974). That is the situation here, so the President must comply with the subpoena. NOTE: If the communications involved military, diplomatic, or national security secrets, then the executive privilege might be absolute, and outweigh law enforcement or judicial needs. U.S. v. Nixon. But that's not the situation here.

John Merrick, the "elephant man," receives disability benefits from the state. The state now wants to terminate those benefits, because it believes he is no longer disabled within the meaning of the disability-benefits statute. Does the state have to give him an evidentiary hearing (at which he can show he is in fact still disabled) before the termination goes into effect?

No. Procedural due process rights attach only when one is deprived of a property or liberty right. Deprivation of one's disability benefits is a right that does qualify for protection. Determining the amount of protection necessary requires balancing the importance of the property or liberty interest in question and the risk an erroneous deprivation in a particular procedure would create (considering the probable value of any additional safeguards) against the importance to the government of the function in question, and the administrative and fiscal expense of a particular safeguard. The individual's interest in disability benefits is not considered "vital" (unlike welfare payments), as they are not based on need; there is little risk of error, as the determination is based on medical evidence, which can be presented in document form without requiring oral testimony; and the cost of full hearings would severely burden the government. Thus, prior notice, the opportunity for a written response, and a posttermination evidentiary hearing are together sufficient to satisfy procedural due process requirements. Mathews v. Eldridge, 424 U.S. 319 (1976).

Commerce Clause

The clause in the Constitution (Article I, Section 8, Clause 1) that gives Congress the power to regulate all business activities that cross state lines or affect more than one state or other nations.

Bush v. Gore

Use of 14th Amendment's equal protection clause to stop the Florida recount in the election of 2000.

Libby Zbiblenik is the executive in charge of Meaningless Paper Shuffling at City Hall. She has an employment contract mandating that she can only be fired for "cause," including any kind of misfeasance or incompetence. Libby comes to work one day and finds a sinister envelope in her inbox. The envelope contains, along with two McDonald's gift certificates, a letter telling Libby she is to be dismissed two weeks hence, due to the poor quality of her work. The note further tells her she can request a posttermination hearing as to the cause of her dismissal. Has Libby been denied due process?

No. Procedural due process rights attach only when one is deprived of a property or liberty right. Deprivation of public employment, when one can only be terminated for cause, is a property right that qualifies for protection. Determining the amount of protection necessary requires balancing the importance of the property or liberty interest in question and the risk an erroneous deprivation in a particular procedure would create (considering the probable value of any additional safeguards), against the importance to the government of the function in question and the administrative and fiscal expense of a particular safeguard. Mathews v. Eldridge, 424 U.S. 319 (1976). In the case of a job requiring good cause for dismissal, due process requires merely notice beforehand and the opportunity for a posttermination evidentiary hearing (at which she can get back pay and reinstatement if she's successful). An evidentiary hearing beforehand is not part of the required procedural package, because of the ability of back pay and reinstatement to make the job holder whole. Arnett v. Kennedy, 416 U.S. 134 (1974). RELATED ISSUE: Say instead that Libby was fired after speaking out against the mayor at a city council meeting. Then she would be entitled to a hearing before she was fired, because a prior hearing is necessary if the dismissal is based on the exercise of a constitutional right. RELATED ISSUE: Say that Libby didn't have the "for cause" clause in her contract and could be fired at will. Assuming she was fired for something other than exercising a constitutional right (e.g., incompetence), she would not have a property right in continued employment, so she would not be entitled to procedural due process at all.

Title II of the 1964 Civil Rights Act bans discrimination in places of public accommodation that serve interstate travelers or buy products if a substantial portion has moved in interstate commerce. Freddy Cobb owns a restaurant, Gloria's, in the State of Old Miss, and he refuses to serve African Americans. Freddy claims the Civil Rights Act is unconstitutional as applied to him, because his restaurant is far away from any interstate and no appreciable part of his business is made from interstate travelers. He does, however, purchase about half his products from a supplier that in turn bought from a wholesaler in another state. Will Freddy win?

No. Similar facts were present in Katzenbach v. McClung, 379 U.S. 294 (1964), in which the Court upheld the Act as applied to a small local restaurant. The Court reasoned that although the restaurant had a small effect on commerce, similar conduct taken in the aggregate would clearly affect interstate commerce. The Court also noted that although Congress made no specific findings of the impact that restaurant discrimination would have on commerce, a rational basis existed for the necessity of the regulation to protect interstate commerce.

Tammy attends the Saints and Sinners Parochial School. A governmental program provides bus service to and from school for Tammy and her classmates, as well as for children at public schools. Is Tammy's bus transportation a violation of the Establishment Clause of the First Amendment?

No. Such a program's aim and effect are secular, and it does not pose a risk of excessive government entanglement with religion. Therefore, the program is valid.

Is the Commerce Clause the same as the Dormant Commerce Clause?

No. The Commerce Clause is in the text of Article I, §8 and authorizes Congress to regulate interstate commerce. The Dormant Commerce Clause is the negative implication of the Commerce Clause: if Congress has power over interstate commerce, the states do not. State regulation of interstate commerce can be a violation of the Dormant Commerce Clause.

Does the Eleventh Amendment bar the federal government from suing a state?

No. The Eleventh Amendment essentially forbids federal court actions against states for money damages, based on the state's past conduct, where the money damages will be payable from state funds. (Even in this situation, it doesn't apply if the state consented or if Congress authorized the suit.) Edelman v. Jordan, 415 U.S. 651 (1974). The following kinds of suits are not forbidden by the Eleventh Amendment: Suits by the federal government or other states against a state; Suits in which injunctive/declarative relief is sought against state officials for federal law violations; Suits by anyone against subdivisions of states (e.g., cities and counties); Suits looking for prospective relief instead of damages for past conduct; Suits in which the claim is that a state official violated federal law (constitutional or otherwise), not state law; Suits brought against a state in state court (although a parallel principle of state immunity may protect a state if sued in its own courts on a federal right); Damages sought against state officials personally (it's damages payable by the state that are barred); and Suits under federal statutes enacted under §5 of the Fourteenth Amendment (which gives Congress the power to enforce the Fourteenth Amendment).

The State of Bitterroot denies convicted felons the opportunity to vote. Does this violate equal protection?

No. The Fourteenth Amendment specifically allows states to disenfranchise felons. Richardson v. Ramirez, 418 U.S. 24 (1974).

The State of Mission Law School accepts 350 of its 3,500 applicants. One of its goals in admitting those students is to build a student body of diverse backgrounds and interests. Mission Law School has a particular commitment to racial diversity, and works to include students from groups who have faced historical discrimination, namely African Americans, Hispanics, and Native Americans. The school's goal is to have a critical mass of unrepresented minority students. To promote this goal of diversity, the Law School's admissions staff reviews individual files carefully with diversity in mind. In this review, while using race as a "plus factor," the admissions staff also considers other possible bases for diversity, such as geographical location and life experience. Bobbie Rudder, a white student with a higher grade point average and LSAT score than some of the accepted minority students, challenges the program as a violation of equal protection. Will her challenge succeed?

No. The Law School program survives strict scrutiny. All racial classifications are subject to strict scrutiny, which means that they are constitutional only if they are narrowly tailored to further compelling governmental interests. In Grutter v. Bollinger, 539 U.S. 306 (2003), the Court recognized diversity as a compelling government interest, in the educational setting, that could justify the use of racial classifications. The Court then ruled that the University of Michigan Law School's program, similar to the hypothetical here, was narrowly tailored because the program did not establish a quota, and used a flexible file review that considered each applicant as an individual.

Miller v. Schoene

The court held that an act that allowed the destruction of red cedars to prevent the spread of cedar rust disease, was not a taking because there is no taking when the government destroys property to preserve another class of property that is of greater value to the public.

In response to numerous accounting scandals, Congress sets up the Public Company Accounting Oversight Board (PCAOB). The members are to be selected by the SEC, an independent federal agency of members appointed by the President. The President's removal power of the SEC is limited except for cause. In turn, the SEC agency was prevented from removing the inferior PCAOB members they selected, except for cause. Is this two-levels-of-protection-from-the-President scheme constitutional?

No. The PCAOB members are inferior federal officers because they have significant authority under federal law, and are not supervised directly by the President, but by someone appointed by the President (SEC). Usually when Congress limits the removal of an inferior officer for cause the President can show disapproval of the officer by removing the agency head that appointed them. The multilevel good-cause protection (SEC member and PCAOB member) was held by the Roberts Court in a 5-4 vote to "subvert[] the President's ability to ensure that the laws are faithfully executed." Free Enterprise Fund v. Public Co. Accounting Oversight Bd., 130 S. Ct. 3138 (2010). NOTE: Removal of the SEC members limited to good-cause is permissible under the Humphrey's Executor principle, because the Commission is an independent quasi-legislative agency.

President Cheap Skate receives a detailed bill passed by both Houses of Congress. The bill funds numerous programs, sending money for flood relief to some parts of the country and funding for farmers to other states. President Skate reads through the bill and decides that farm programs in the Northeast are overfunded compared to the rest of the nation. He takes out a big black Magic Marker and crosses out the lines of the bill that give money to vegetable farms in Noo Yawk and Road Isle. Then he signs the bill so that the rest of the bill's provisions become law. Is the President's action constitutional?

No. The President has attempted a line-item veto. The Court has ruled that line-item vetoes violate Separation of Powers even when they are authorized by congressional legislation. Clinton v. City of New York, 524 U.S. 417 (1998).

Does the Supremacy Clause state that the Supreme Court is supreme over the other branches in interpreting the Constitution?

No. The Supremacy Clause states that the Constitution and Laws of the United States are supreme over state laws. NOTE: In Cooper v. Aaron, 358 U.S. 1 (1958), the Court stated "the federal judiciary is supreme in the exposition of the law of the Constitution."

Is the Bill of Rights as a whole "incorporated" into the Due Process Clause of the Fourteenth Amendment?

No. The Supreme Court has only made applicable to the states those provisions in the Bill of Rights it deems essential to the "American scheme of justice." Although this covers most of the Bill of Rights, the Court has refused to "incorporate" certain provisions, including the right to a grand jury.

The State of Surprise grants the Questionable Utility Co. a monopoly on business in the state. The state heavily regulates the utility. When the company cuts off the electricity to Ima Deadbeat's house without notice and a hearing, is "state action" involved?

No. The actions of a private entity only rise to the level of state action when the state either provided a "mantle of authority" for the actions, or was significantly involved in, or encouraged, those actions. One common way for this to happen occurs when the state delegates to a private actor "powers traditionally exclusively reserved to the State." However, this phrase has been interpreted extremely narrowly by the Court; only the running of elections and the furnishing of a complete set of municipal functions ("company town") have qualified. The Court has held that a utility's furnishing of a vital service (e.g., electricity) on a tightly regulated, monopoly basis does not constitute the requisite state action. Therefore, the actions of the utility are not subject to Fourteenth Amendment requirements. Jackson v. Metropolitan Edison Co., 419 U.S. 345 (1974).

The State of Pearlygate enacts a statute requiring public schools to teach "creation science" theory whenever the theory of evolution is taught. The stated intent is to protect academic freedom; however, the legislative history indicates that the law is designed to advance the viewpoint that a supernatural being created humankind. Would the statute survive a First Amendment challenge?

No. The government must be "religion neutral." A statute that advances religious interests is only deemed to be religion neutral if it passes this three-part test from Lemon v. Kurtzman, 403 U.S. 602 (1971): (1) It must have a secular purpose; (2) It must not have as its principal or primary effect the advancement of religion; and (3) It must not foster excessive government entanglement with religion. The problem here is the "secular purpose" requirement. The legislative history shows that the intent is to advance the belief that a supernatural being created humankind, and that is a religious purpose. As a result, the statute violates the Establishment Clause of the First Amendment. Edwards v. Aguillard, 482 U.S. 578 (1987).

Can Congress alter the Supreme Court's original jurisdiction?

No. The grant of original jurisdiction in Article III, §2 is self-executing; however, Congress may grant concurrent jurisdiction to lower federal courts. That is, Congress may give the lower federal courts the right to hear the same kind of cases (e.g., cases in which a state is a party) as fall within the Supreme Court's original jurisdiction. NOTE: Congress has the power to change federal appellate jurisdiction, as long as the change is made in a neutral manner, without intent to decide the merits of the case through the change in jurisdiction. U.S. v. Klein, 80 U.S. 128 (1872).

Kramer v. Union Free School District

Voting

Grant Vizier owns a garment factory that makes robes used in Ku Klux Klan functions. Every July 4th, he opens his fabulous estate to his 500 employees and their guests for a barbecue and a fireworks display. One year one of his employees, Thomas Jefferson, brings a non-white date, Sally Hemings. Vizier orders her off the property because of her race. She refuses to leave. Vizier files a criminal trespass complaint against her. Would her conviction violate the Fourteenth Amendment?

No. The issue here is whether there is state action involved in the court's convicting Sally of trespass when she was ordered off Vizier's property for racially discriminatory reasons. One-way private conduct can constitute state action is for the state to lend judicial support for the private conduct; that's what happens when a court enforces a racially restrictive covenant. However, all that's happening here is that the court is enforcing a race-neutral trespass law; judicial enforcement is forbidden only when the rule of law being enforced is, racially speaking, non-neutral. Because the rule allowing exclusivity of private property is a racially neutral one, a private person may refuse to allow people onto his property on racial grounds as long as he is not otherwise connected with state action. As a result, Sally's conviction would be constitutionally permissible.

In determining whether the Supreme Court will review a case, does it matter whether the lower court that decided the case is a state court or a federal court?

No. The most important thing to remember about Supreme Court review of state court decisions is that there must be an issue of federal law—the Supreme Court can't review state court decisions adjudicating only state law issues. There's no limitation like this for discretionary review of lower federal cases.

Congress passes a statute directing the Secretary of Transportation to withhold 5% of federal highway funds from states with a drinking age younger than 21. The purpose of the statute is to reduce the number of accidents created by young people driving between states to take advantage of lower drinking ages. The State of Wildwest challenges the statute, claiming it's an impermissible "string" on a federal grant. Is it? (Assume the Twenty-First Amendment would bar Congress from establishing a national minimum drinking age.)

No. The power Congress is exercising here is its general welfare power—that is, Congress may tax and spend for the general welfare. Incident to its power to spend, Congress may attach conditions to the receipt of federal funds, as long as: It does so unambiguously, allowing the states to exercise their choice knowingly (thus, the pressure cannot be so great as to be coercive); The condition is related to a national concern; and The condition is not independently barred by another constitutional provision—that is, the power may not be used to induce the states to engage in activities that would themselves be unconstitutional. Here there's no coercion, because only 5% of the state's highway funds are at risk; the concern here is safe interstate travel, which is a concern of general welfare. As to the third condition, a state drinking age of 21 wouldn't violate anyone's constitutional rights. Thus, the condition is valid. South Dakota v. Dole, 483 U.S. 203 (1987). NOTE: The South Dakota case assumed the Twenty-First Amendment would bar a national minimum drinking age, but didn't decide the issue. Note that this means through its spending power Congress may indirectly accomplish things it couldn't do directly (e.g., establish a nationwide minimum drinking age). NOTE: If Congress, through the conditional spending power, induced the states to pass laws that would themselves violate constitutional rights of individuals, those congressional actions would be unconstitutional.

The State of Suspicion enacts a statute requiring that the race of candidates for public office be listed on ballots. Alf, a Melmackian running for sheriff, challenges this requirement on equal protection grounds. Because the statute treats all candidates alike, regardless of their race, will the statute be valid?

No. The statute is discriminatory, and it violates equal protection. To violate equal protection, the government must purposefully discriminate—that is, discriminatory impact alone isn't sufficient. A statute can be purposefully discriminatory on its face, in its administration, or due to a discriminatory motive or intent. The statute here has a discriminatory intent, in that listing race on a ballot would certainly support, and perhaps encourage, racial prejudice. As purposeful discrimination based on race, the statute must be necessary to support a compelling interest to be valid. Because there is no legitimate interest in fostering prejudice, the statute is unconstitutional. Anderson v. Martin, 375 U.S. 399 (1964). COMPARE: Recording the race of those seeking a divorce would be valid if records are kept for statistical purposes, as there would be no discriminatory intent (or impact) involved in keeping such records.

Congress passes the Gambling Act, which provides that any state that allows non—Native American gambling must negotiate with any Native American tribes in the state to give them an opportunity to conduct comparable gaming operations. The Gambling Act authorizes lawsuits by the Native American tribes in federal courts if the states fail to negotiate with the tribes in good faith. Native American Tribe sues the State of Chance in federal court alleging that the State did not negotiate in good faith. Can the federal court hear the lawsuit?

No. These facts are based on the case of Seminole Tribe of Florida v. Florida, 517 U.S. 44 (1996), in which the Court held that Congress could not use its Article I powers to abrogate the state's Eleventh Amendment immunity. Seminole Tribe was a 5-4 decision that started the Court's line of cases limiting Congress' ability to abrogate state sovereign immunity. NOTE: Art. I, §8, cl. 3 gives Congress the authority to regulate commerce with the Native American tribes. This is part of the Commerce Clause of Article I, §8.

Would a person's yelling at someone "You *******ed Fascist" be speech protected by the First Amendment?

No. These particular words have been held to be "fighting words," and fighting words are not protected by the First Amendment. Chaplinsky v. New Hampshire, 315 U.S. 568 (1942). Fighting words are personally abusive epithets likely to incite immediate physical retaliation in the ordinary citizen. The public interest in keeping order outweighs any slight social value of the expression. Id. NOTE: The Court has not upheld a conviction on fighting words since Chaplinsky, more than 60 years ago, and it is unlikely that a fighting words conviction would survive today.

Is there a constitutional requirement that states outlaw discrimination?

No. They are only forbidden from authorizing or encouraging it.

The President wants to appoint Judge Torquemada to the U.S. Supreme Court. Torquemada presents himself as an everyday conservative, but an investigation into his judicial background shows that he's known as the "Grand Inquisitor" for his brutal and imperious style. Can the President appoint Torquemada without the consent of the Senate?

No; he may appoint judges (and other top-level federal officers, like Cabinet members) only on the advice and consent (majority vote) of the Senate. Art. II, §2.

New York City Transit Authority v. Beazer

The court held that the entity's rule, which refused to employ anyone who used methadone, was constitutional because the classification was rationally related to the state's interest in protecting public safety and anything short of total exclusion would have been insufficient.

For the last 100 years, the City of Alphaquad has used an at-large election system to elect its five-member governing body, called "the Federation." The city's population is 52% human and 48% Vulcan; however, the at-large electoral system has prevented any Vulcan from ever being elected to the Federation. Although there's no discriminatory purpose evidenced by the system itself, and it wasn't adopted for that purpose, the city has been unresponsive to the needs of its Vulcan community. Would the at-large system survive an equal protection challenge?

No. To violate equal protection, discrimination must be purposeful—that is, discriminatory impact, standing alone, doesn't make a governmental action invalid. A statute can be purposefully discriminatory on its face, in its administration, or due to a discriminatory motive or intent. The issue in these facts is that what you've got is a significant impact—no Vulcan has ever been elected to the Federation—which isn't enough, by itself, to make the at-large system discriminatory. However, discriminatory impact can be evidence of an improper motive, and this, coupled with the City's unresponsiveness to Vulcan needs, would be sufficient to evidence purposefulness. A purposefully discriminatory governmental action based on race must be necessary to promote a compelling governmental interest to be valid. Because discriminating against minorities is not a legitimate governmental interest, the at-large system is unconstitutional. Rogers v. Lodge, 458 U.S. 613 (1982).

Congress passes legislation creating a Federal Election Commission (FEC). The legislation provides that a majority of voting members of the FEC will be appointed by the President pro tem of the Senate and the Speaker of the House of Representatives. Is this selection procedure constitutional?

No. Under Article II, §2, cl. 2, Congress has three options in delegating the appointment of "lower federal officers" (i.e., non-top-level officers) whose tasks are executive in nature. It may vest the appointment of such officers in: (1) The President alone; (2) The courts of law; or (3) The heads of departments (i.e., Cabinet members). Here, the legislation calls for appointment by the President pro tem of the Senate and the Speaker of the House, neither of whom fits these three categories. Thus, the selection procedure is unconstitutional. Buckley v. Valeo, 424 U.S. 1 (1976). NOTE: The Buckley Court defined federal officers as appointees "exercising significant authority pursuant to the laws of the [United States]." NOTE: Congress itself does not have the power to appoint lower federal officers; this power must be delegated as outlined above. As to top-level federal officials—ambassadors, Cabinet members, Supreme Court justices, etc.—the President has the power to appoint, but only upon the "advice and consent" (i.e., majority vote in favor) of the Senate. Article II, §2, cl. 2 (the Appointments Clause). NOTE: While Congress can't appoint executive officers, it can set the qualifications for such appointments (e.g., minimum age and experience requirements), which can narrow the field of potential candidates considerably.

Congress passes the Superfund Extension Act of 2009, which bars certain types of pollution, and says that violators may be sued for money damages in federal court by "any affected individual." The Act expressly says that states are subject to its provisions. Harry Homeowner brings a damage suit against the State of Old Jersey in federal court, alleging that an airfield owned and operated by the state has dumped jet fuel onto the adjacent property (owned by Harry) in violation of the Act. Assuming that the state has in fact violated the Act, may Harry recover?

No. What Congress tried to do in making the Act applicable to the states as defendants was to cause the Eleventh Amendment not to apply. But except for statutes enacted by Congress pursuant to its special powers to enforce the Thirteenth, Fourteenth, and Fifteenth Amendments, or the Bankruptcy Clause of Art. I, §8, Congress has no power to abrogate the Eleventh Amendment. Seminole Tribe of Fla. v. Fla., 517 U.S. 44 (1996) (when Congress acts under the Commerce power, it cannot abrogate the Eleventh Amendment). The Act here cannot be justified under any of these post-Civil War Amendments, and would instead have to be supported by the Commerce power. Therefore, the states still have their Eleventh Amendment immunity despite what Congress says. Because Harry is trying to hold a state liable for money damages in federal court, the Eleventh Amendment applies, giving the state immunity from his suit. NOTE: The Seminole Tribe case means that the states are immune from private money-damage suits in federal court brought under federal copyright statutes, trademark statutes, environmental laws, and any other statutes that are not supported by the Thirteenth, Fourteenth, and Fifteenth Amendments, or the Bankruptcy Clause of Art. I, §8. N.B.: The Court has held that, unlike other sections of Art. I, the Bankruptcy Clause authorizes Congress to abrogate state sovereign immunity. Central Va. Comty. Coll. v. Katz, 546 U.S. 356 (2006). NOTE: On the other hand, the Eleventh Amendment would not protect Old Jersey against (1) a private suit by Harry for an injunction against further violations (as opposed to a suit for money damages) or (2) a civil suit for a fine or damages brought by the federal government—the Amendment applies only to federal court suits brought by private parties seeking damages against a state.

The City of Brotherhood has an ordinance requiring that contractors on city construction projects set aside 30% of their subcontracts for minority-owned businesses. The ordinance was enacted after a city council study showed that only ½ of 1% of city contracts were awarded to minority-owned businesses, despite a 50+% minority population. (There's no evidence that this paucity of minority contracts is due to past discrimination by the city.) Will the ordinance withstand an equal protection attack?

No. Where a state action creates a race-based classification, the strict scrutiny test will determine the action's validity, even if the action is designed to remedy past discrimination. Richmond v. J. A. Croson, 488 U.S. 469 (1989). Thus, the action must be necessary to promote a compelling state interest. A government entity does have a compelling interest in remedying its own past discrimination (whereas there is no compelling interest in the traditional form of discrimination against minorities). The problem here is that there's no evidence of Brotherhood's own past discrimination, so there's no compelling interest. The relevant statistic would be the number of qualified minority contractors, not the percentage of contracts awarded to minorities, which is insufficient evidence of past racial discrimination to justify a race-based remedy. (For instance, if there were an inordinately small number of qualified minority-owned contractors, the of 1% figure would not indicate discrimination.) Richmond v. J.A. Croson, 488 U.S. 169 (1989). Even given that figure, the program here would not be considered necessary, as it's insufficiently narrowly tailored to withstand strict scrutiny. Note that rigid racial "quotas," like the one here, are virtually never upheld. Richmond v. J.A. Croson, 488 U.S. 169 (1989).

Lucas v. South Carolina Coastal Council

The court held the state law that prevented plaintiff from constructing homes on his coastal lots because they were located in a coastal zone, constituted a taking requiring just compensation because there was no reasonably economically viable use of the property.

The City of Whoopee allows only commercial messages to be displayed on its billboards; no non-commercial messages are allowed. Is this regulation constitutionally valid?

No; it is an unconstitutional violation of the freedom of speech. Metromedia v. City of San Diego, 453 U.S. 490 (1981). Billboards are a public forum. As a result, they are subject to time, place, and manner restrictions, which will be valid only if they are content neutral and narrowly tailored to serve a substantial governmental interest, and if there are alternative channels of communication available. The problem here is that the regulation isn't content neutral; it prohibits non-commercial messages while allowing commercial ones. As a result, the regulation could only be valid if it involved a category of non-protected speech (e.g., obscenity) or it survived strict scrutiny (i.e., it was necessary to achieve a compelling state interest and was narrowly tailored to achieve that interest). The regulation here does not satisfy either condition. As a result, the statute is unconstitutional.

The State of Minds has a state law that requires a person to be an in-state resident for at least one year to vote in state elections. The statute is attacked on equal protection grounds. Is it valid?

No; it is an undue burden on the right to interstate travel and the right to vote. The statute creates a classification involving the right to exercise two fundamental rights—the right to travel interstate and the right to vote. Thus, the classification must be necessary to a compelling state interest. States have a valid interest in restricting voting to their own citizens to prevent voting fraud and promote a knowledgeable electorate. However, the test of validity must consider less burdensome alternatives. That's the problem here; the duration of the residency requirement. One year is too long. Dunn v. Blumstein, 405 U.S. 330 (1972). RELATED ISSUE: The Supreme Court has held that a 50-day residency requirement is valid (and may be close to the upper limit). Marston v. Lewis, 410 U.S. 679 (1973).

Do states have any power to regulate foreign commerce?

No; regulating foreign commerce is exclusively a federal power. RATIONALE: There must be only one voice in regulating commercial relations with other governments. Michelin v. Wages, 423 U.S. 276 (1976). RELATED ISSUE: A state can only tax foreign commerce if these six requirements are met: (1) The tax is non-discriminatory; (2) The activity being taxed has sufficient "minimum contacts" with the taxing state; (3) The tax is apportioned fairly; (4) The tax is related to services provided by the state; (5) The tax notwithstanding apportionment does not create a substantial risk of international multiple taxation; and (6) The tax doesn't prevent the federal government from speaking "with one voice when regulating commercial relations with foreign governments." Japan Line, Ltd. v. Los Angeles, 441 U.S. 434 (1979).

The State of Osage intends to hold an election to vote for officers of a water storage district. It intends to restrict the right to vote to landowners in the district and to weigh the votes by acreage owned. Is this a violation of the "one person, one vote" requirement?

No; the right to vote can be limited to a select group of people when the governmental unit for which elections are being held has a limited purpose that disproportionately affects only one group. Here, the landowners in the district receive all the benefits and bear all the burdens of the election (it's for officers of their water district, which assures their water supply). Thus, the restriction on voting is valid. Salyer Land Co. v. Tulare Lake Basin Water Storage District, 410 U.S. 719 (1973). COMPARE: Requiring land ownership to vote for school board members violates equal protection, as the classification is "irrelevant" to the state's objectives. Kramer v. Union Free School District, 395 U.S. 621 (1969). RELATED ISSUE: Requiring property ownership by candidates also violates equal protection. Turner v. Fouche, 396 U.S. 346 (1970).

The Food and Drug Administration establishes standards for the collection of blood plasma. Transylvania County in the State of Silvercross has an ordinance that imposes additional regulations on collecting blood plasma. Dracula Enterprises, a company that collects blood plasma, claims the federal standard preempts the county standard. Does it?

No; the state statute is valid. The issue here is whether the federal standard preempts the field of blood plasma collection, as there's no direct conflict between the two statutes. If a subject area is one that has traditionally been subject to mainly local rather than national regulation, preemption is unlikely. Here, the field is one historically left to the states—health and safety regulation. Thus, a court would be unlikely to find preemption (absent a clear expression from Congress that it intended to preempt the entire area). Hillsborough County, Fla. v. Automated Medical Labs, 471 U.S. 707 (1985). RELATED ISSUE: The converse is also true; a federal statute in an area of historically national control (e.g., patents, trademarks, immigration, bankruptcy) will generally preempt a related state statute.

Reynolds v. Sims

Voting 14th amendment requires state legislative districts reflect fair "one person, one vote" rule

Stringer Ball (an African American) challenges a state prison policy segregating new inmates for 60 days according to their race because he wants to be cellmates with his best friend, Jimmy McNutty (a Caucasian). The state defends the policy as race neutral and necessary to prevent violence from race gangs. Who will win?

Not clear. The one thing that is clear is that strict scrutiny will apply, as it will for all racial classifications imposed by the government. In Johnson v. California, 543 U.S. 499 (2005), the Court remanded a prison segregation case to determine whether the segregation could survive strict scrutiny. The Court observed that prison segregation could increase racial hostility, but also noted that prisons are dangerous places, and policies such as racial segregation may survive a strict scrutiny analysis as necessary to promote the compelling interest of safety of officers and prisoners.

Congress enacts a national marriage tax of $10,000 that must be paid whenever a couple is married. Some members of Congress explained that they supported the tax because they thought individuals married without sufficient preparation and they wanted to encourage couples to wait and think over the obligations of marriage before taking their vows. Is the marriage tax constitutional?

Not if it is clear that Congress is trying to dissuade marriage rather than to collect taxes. If the tax is a "disguised regulation" it may be declared unconstitutional. NOTE: Courts are likely to sustain regulatory taxes if Congress would have the authority under another enumerated power like the Commerce Clause to pass the legislation. N.B.: If a tax produces substantial revenue it is more likely to be upheld even if it has regulatory aspects.

Does Congress' "general welfare" power give it the power to enact any legislation necessary for the "general welfare" of the nation?

No—it's considerably narrower than that. Congress' "general welfare" power refers solely to the power to provide for the general welfare by imposing taxes (e.g., Social Security tax) and spending money. NOTE: Such taxing and spending must be done for the general welfare; thus, for instance, Congress couldn't levy a tax on the sale of gasoline nationwide and give the proceeds to a single oil company to help it find ways to increase its profitability. NOTE: Congress could use its spending power to achieve indirectly objectives it couldn't achieve directly pursuant to its regulatory power, as long as doing so isn't coercive or doesn't violate an independent constitutional limitation (like the Bill of Rights). So Congress may spend and tax for the general welfare, but it has no power to regulate to provide for the general welfare. To regulate, Congress must be acting pursuant to some enumerated power (e.g., the Commerce Clause).

Illegitimacy, like gender, is a "quasi-suspect" class, subjecting it to intermediate scrutiny.

One important point to look for is the purpose behind a statute tied to illegitimacy; thus, courts won't uphold discriminatory laws intended to punish illegitimate children. For instance, a state law prohibiting illegitimate children from pressing wrongful death actions concerning their parents' death, while allowing wrongful death claims by legitimate children, would be invalid. Levy v. Louisiana, 391 U.S. 68 (1968).

Under what circumstances will a party have standing to vindicate someone else's rights (i.e., third-party standing)?

Only where the plaintiff has suffered direct injury as a result of the third party's constitutional or federal rights being violated, and only if: The third party may not be able to vindicate her own rights; and There is a special relationship between the plaintiff and the third party. NOTE: One type of third-party standing is the standing of a state to sue another state on behalf of its citizens as parens patriae, to protect its citizens' comfort, health, and property rights (including environmental damage cases). Note that this covers suits against other states, not the federal government. NOTE: An association will have standing on behalf of its members if the following three requirements are satisfied: The members would have standing; The interests the association seeks to protect relate to the association's purpose; and Neither the claim itself nor the relief requested requires the individual participation of association members. Hunt v. Washington Apple Advertising Comm'n, 432 U.S. 333 (1977).

Advisory Opinions (Con Law)

Opinion providing no redressability. Prohibited.

The State of Security requires that people seeking a driver's license must provide a government photo where the individual is not wearing anything that covers the face. Lucy is a member of the Church of Modesty, which forbids its members to appear in public or be photographed with their faces uncovered. She challenges the driver's license photo requirement as an undue burden on her religious practice. Will she win?

Probably not. The photo requirement for a driver's license is a valid and neutral law of general applicability. As long as the government's requirement has an unintentional effect of burdening religion and is not aimed specifically at disfavoring a particular religious practice it will be upheld. NOTE: Plaintiffs may pursue religious freedom claims under state statutes (RFRAs) that allow lawsuits about laws that substantially burden religion that do not meet the compelling state interest test. The federal government may be challenged under the federal RFRA statute. Boerne v. Flores, 521 U.S. 507 (1997).

Murr v. Wisconsin

Regulatory restrictions on property rights to a taking depend on their impact on the "parcel as a whole"

Whitney v. California

Restrictions on freedom of speech

Kassel v. Consolidated Freightways Corp.

STATE REGULATION OF INTERSTATE COMMERCE

West Lynn Creamery, Inc. v. Healy

STATE REGULATION OF INTERSTATE COMMERCE

14th Amendment

Section 1. All persons born or naturalized in the United States, and subject to the jurisdiction thereof, are citizens of the United States and of the state wherein they reside. No state shall make or enforce any law which shall abridge the privileges or immunities of citizens of the United States; nor shall any state deprive any person of life, liberty, or property, without due process of law; nor deny to any person within its jurisdiction the equal protection of the laws. Section 2. Representatives shall be apportioned among the several states according to their respective numbers, counting the whole number of persons in each state, excluding Indians not taxed. But when the right to vote at any election for the choice of electors for President and Vice President of the United States, Representatives in Congress, the executive and judicial officers of a state, or the members of the legislature thereof, is denied to any of the male inhabitants of such state, being twenty-one years of age, and citizens of the United States, or in any way abridged, except for participation in rebellion, or other crime, the basis of representation therein shall be reduced in the proportion which the number of such male citizens shall bear to the whole number of male citizens twenty-one years of age in such state. Section 3. No person shall be a Senator or Representative in Congress, or elector of President and Vice President, or hold any office, civil or military, under the United States, or under any state, who, having previously taken an oath, as a member of Congress, or as an officer of the United States, or as a member of any state legislature, or as an executive or judicial officer of any state, to support the Constitution of the United States, shall have engaged in insurrection or rebellion against the same, or given aid or comfort to the enemies thereof. But Congress may by a vote of two-thirds of each House, remove such disability. Section 4. The validity of the public debt of the United States, authorized by law, including debts incurred for payment of pensions and bounties for services in suppressing insurrection or rebellion, shall not be questioned. But neither the United States nor any state shall assume or pay any debt or obligation incurred in aid of insurrection or rebellion against the United States, or any claim for the loss or emancipation of any slave; but all such debts, obligations and claims shall be held illegal and void. Section 5. The Congress shall have power to enforce, by appropriate legislation, the provisions of this article.

13th Amendment

Section 1. Neither slavery nor involuntary servitude, except as a punishment for crime whereof the party shall have been duly convicted, shall exist within the United States, or any place subject to their jurisdiction. Section 2. Congress shall have power to enforce this article by appropriate legislation.

15th Amendment

Section 1. The right of citizens of the United States to vote shall not be denied or abridged by the United States or by any state on account of race, color, or previous condition of servitude. Section 2. The Congress shall have power to enforce this article by appropriate legislation.

When does a plaintiff have standing to pursue a claim in federal court?

Standing requires that the plaintiff establish: INJURY: plaintiff has a concrete injury that is actual or imminent; CAUSATION: the injury is traceable to the defendant's conduct; and REDRESSABILITY: the injury can be redressed by the federal court (e.g., by striking down the offending statute). Lujan v. Defenders of Wildlife, 504 U.S. 555 (1992). As a general matter, look to see if the claimant in question has a personal stake in the outcome of the case. NATURE OF INJURY REQUIRED: It can be economic, aesthetic, environmental, recreational, or reflect a "spiritual stake" in the First Amendment rights of free exercise and establishment. In addition, it must not be so indeterminate or diluted that its impact on the plaintiff is no different from the injury to citizens at large. Such a widespread injury is called a "generalized grievance" and is not heard by the courts. EXCEPTIONS: There are several exceptions to the general standing requirements. The most significant ones include third-party standing, taxpayer standing, and congressionally created standing, which we'll discuss in more detail on other cards. NOTE: As a practical matter, the Court uses standing as a tool to avoid deciding certain cases on the merits.

Standing

State level: A state's statutes will determine what constitutes standing in that particular state's courts. These typically revolve around the requirement that plaintiffs have sustained or will sustain direct injury or harm and that this harm is redressable. Federal level: At the federal level, legal actions cannot be brought simply on the ground that an individual or group is displeased with a government action or law. Federal courts only have constitutional authority to resolve actual disputes

RECAP: ANALYZING STATE STATUTES AFFECTING INTERSTATE COMMERCE

State regulations affecting interstate commerce must satisfy the following three requirements: The regulation must pursue a legitimate state end; The regulation must be rationally related to that legitimate state end; and The state's interest in the regulation must outweigh the regulatory burden.

What are the three levels of scrutiny courts use in analyzing equal protection problems?

Strict scrutiny, intermediate scrutiny, and the rational relation test.

Williams v Rhodes

Voting States cannot prohibit 3rd parties from forming and participating in the election process.

The Encounters Construction Co., a Caucasian-owned construction company, submits the lowest bid for a subcontract to construct telescopes for a UFO-watching tower in rural Nevada. However, the general contractor accepts a bid from E.T. Ltd., a minority-owned firm qualifying under congressionally authorized regulations as a Disadvantaged Business Enterprise. The prime contractor awards the contract to E.T., and for doing so receives a financial incentive from the federal government. Encounters Construction brings suit. What type of scrutiny will this federal regulation have to meet?

Strict scrutiny. Congressionally authorized, race-conscious, affirmative action programs are subject to strict scrutiny, just as such programs are scrutinized when enacted by state or local governments. The regulation at issue here can be upheld only if it was necessary to achieve a compelling state interest. Adarand Construction, Inc. v. Pena, 515 U.S. 200 (1995). The court must consider: (1) whether the governmental interest being served is compelling; (2) whether the racial classification is narrowly tailored to serve that interest (i.e., whether race-neutral means might have been effective to achieve the interest); and (3) whether the remedy is acceptably short-lived (i.e., whether it avoids lasting longer than the discriminatory effects it is designed to eliminate). Here, the provision will probably be struck down, because it's rare that direct government financial incentives rendered on a race-conscious basis will be found to be a "necessary" means of wiping out past discrimination.

Goss v. Lopez

Students have due process rights and cannot be suspended without a hearing.

Lochner v. New York

Supreme Court case that decided against setting up an 8 hour work day for bakers

Mathews v. Eldridge

Supreme Court case that specified due process considerations of probation supervision.

Washington v. Davis

Supreme Court ruled that a test that has an adverse impact against a protected class is legal if it is job-related.

Garcia v. San Antonio Metropolitan Transit Authority

THE TENTH AMENDMENT AS A FEDERALISM-BASED LIMITATION ON CONGRESSIONAL POWER

Gitlow v. New York

The 1925 Supreme Court decision holding that freedoms of press and speech are "fundamental personal rights and liberties protected by the due process clause of the Fourteenth Amendment from impairment by the states" as well as by the federal government.

Roe v. Wade

The 1973 Supreme Court decision holding that a state ban on all abortions was unconstitutional. The decision forbade state control over abortions during the first trimester of pregnancy, permitted states to limit abortions to protect the mother's health in the second trimester, and permitted states to protect the fetus during the third trimester.

McCleskey v. Kemp

The 1987 Supreme Court decision that upheld the constitutionality of the death penalty against charges that it violated the Fourteenth Amendment because minority defendants were more likely to receive the death penalty than were White defendants.

Morrison v. Olson

The Court held that Congress's "good cause" removal provision of the Ethics in Government Act was constitutional, since it does not interfere with the President's ability to perform his constitutional duties.

Palazzolo v. Rhode Island

The Court held that even when a person takes title to property knowing that it is subject to a restriction, they may still bring a takings claim.

City of Philadelphia v. New Jersey

The Court held that the state law prohibiting the importation of waste from out-of-state was a violation of the dormant commerce clause because the law was facially discriminatory.

Lujan v. Defenders of Wildlife

The Court held the plaintiffs lacked standing under the Endangered Species Act to challenge actions regarding the protection of endangered species in Egypt, since they had no concrete or particularized injury.

Pennsylvania Coal v Mahon

The Court held the state law forbidding the company from mining coal under the land constituted a taking because private property is considered taken when the regulation results in a severe diminution in the value of the property.

Exxon Corp. v. Governor of Maryland

The Court held the state law prohibiting oil companies from operating gas stations within the state was not a violation of the dormant commerce clause because it did not impermissibly burden interstate commerce. The dormant commerce clause protects the interstate market, not particular interstate companies.

Printz v. US

The Court invalidated a federal law that required local police to conduct background checks on all gun purchasers

Sunnyvale grants funds to organizations with playgrounds whose lights are powered exclusively through solar energy. Son Rise Church (SRC) installed solar panels for its playground and then applied for the grant. Sunnyvale denied SRC's grant because its state's constitution forbids the use of money in the public treasury to aid any church. SRC sues and argues the denial violated the First Amendment's Free Exercise Clause and the Fourteenth Amendment's Equal Protection Clause. What will the Court decide?

The Court will find the denial of the grant—a neutral and secular aid program as found in Trinity Lutheran Church of Columbia, Inc. v. Comer—violated the First Amendment's Free Exercise Clause because it discriminated against an otherwise eligible organization based solely on its religious nature. As a result, SRC was denied the opportunity to compete for a benefit available to secular organizations, thus burdening religious practice. Without a sufficiently compelling state interest, the policy of excluding churches from these grants fails to meet strict scrutiny. The Justices disagreed about the free exercise reasoning and set up possible new developments allowing more church funding in First Amendment law. Trinity Lutheran Church of Columbia, Inc. v. Comer, 137 S. Ct. 2012 (2017).

Through what provision in the Constitution does a right become "incorporated," and thus, applicable against the states?

The Due Process Clause of the Fourteenth Amendment. Otherwise, a right is not applicable to the states, because the Bill of Rights in general does not apply to the states.

What clause of the Constitution did Justice Scalia rely on in his concurrence in the marijuana case, Gonzales v. Raich, 545 U.S. 1 (2005), in ruling that Congress has the power to regulate home-grown, intrastate medical marijuana?

The Necessary and Proper Clause, Article I, §8, cl. 18. Justice Scalia wrote that Congress can regulate intrastate activity that does not substantially affect interstate commerce if "that regulation is a necessary part of a more general regulation of interstate commerce." Congress could conclude that regulating local marijuana was necessary to the enforcement of the drug laws. NOTE: Congress has authority to pass the Controlled Substances Act because drugs are bought and sold in interstate commerce.

What clause of the Constitution does the line-item veto violate?

The Presentment Clause, Article I, §7, cl. 2, which states that after a bill is approved by both Houses it shall be presented to the President; "if he approve he shall sign it, but if not he shall return it." The line-item veto violates this procedure because the veto occurs after the bill is signed into law and the cancellation applies to only part of the bill. The Presentment Clause requires veto of the entire bill.

Privileges and Immunities Clause

The Privileges and Immunities Clause of Article IV, Section 2 of the Constitution states that "the citizens of each state shall be entitled to all privileges and immunities of citizens in the several states." This clause protects fundamental rights of individual citizens and restrains state efforts to discriminate against out-of-state citizens. However, the Privileges and Immunities Clause extends not to all commercial activity, but only to fundamental rights. --The Federalist Papers also provides some insight into the clause. Madison's Federalist No. 42. Madison stated: "Those who come under the denomination of free inhabitants of a State, although not citizens of such State, are entitled, in every other State, to all the privileges of free citizens of the latter; that is, to greater privileges than they may be entitled to in their own State . . . ." In Federalist No. 80, Hamilton expressed his belief in the clause's importance when he wrote that the Privileges and Immunities Clause (the version in the Constitution) is "the basis of the union."

Name the rights that are "fundamental" for substantive due process purposes.

The Right to Marry; The Right to Custody of One's Children; The Right to Keep the Family Together; The Right to Control Upbringing of Children; The Right to Procreate; The Right to Purchase and Use Contraception; and A Protected Liberty Interest for Competent Adults to Refuse Unwanted Medical Care. RELATED ISSUE: Although abortion was recognized as a fundamental right in Roe v. Wade, the government is now held to the undue burden standard of Casey; a "finding of an undue burden standard is a shorthand for the conclusion that a state regulation has the purpose or effect of placing a substantial obstacle in the path of a woman seeking an abortion of a nonviable fetus." N.B.: The Court has ruled that the right to privacy protects private consensual adult sexual activity, including homosexuality, without identifying this privacy as a fundamental right. The Court invalidated a homosexual sodomy statute in Lawrence v. Texas, 539 U.S. 558 (2003), using rational basis review. Same-sex marriage is also a constitutional right. Obergefell v. Hodges, 135 S. Ct. 2584 (2015). NOTE: Some textbooks treat the punitive damages cases and the Caperton decision about a judge's refusal to recuse from a case involving a campaign contributor as a new type of economic substantive due process.

The State of Enterprise has a law prohibiting interracial marriage. Lieutenant Uhura, who is African American, and Captain Kirk, who is Caucasian, want to marry. They challenge the statute on equal protection grounds. The state argues that because the statute treats all races equally, it's valid. What result?

The State of Enterprise loses. In Loving v. Virginia, 388 U.S. 1 (1967), the Court invalidated a similar statute, stating that any racial classification, even one that treats the races equally, should be subject to strict scrutiny. The Court observed that the law prohibited only interracial marriages involving white persons; non-whites could marry each other. Its purpose was therefore to protect white supremacy, which is a violation of equal protection. RELATED ISSUE: The Court also found a violation of substantive due process in the antimiscegenation statute because it interfered with the fundamental right to marry.

US v. Curtiss-Wright

The Supreme Court concluded not only that foreign affairs power was vested in the national government as a whole but also that the President of the United States had inherent powers in the foreign affairs field that was not dependent upon congressional delegation

US v. Nixon

The Supreme Court does have the final voice in determining constitutional questions; no person, not even the President of the United States, is completely above law; and the president cannot use executive privilege as an excuse to withhold evidence that is 'demonstrably relevant in a criminal trial

Parents Involved in Community Schools v. Seattle School District No. 1

The Synthesis of Brown and Affirmative Action

Define religious belief as it applies to the First Amendment.

The belief must parallel an orthodox religious belief and exceed a purely political or philosophical view. U.S. v. Seeger, 380 U.S. 163 (1965). However, the belief need only be sincere; it need not conform to any particular religious sect. Frazee v. Illinois Dep't of Employment Security, 489 U.S. 829 (1989).

What are the elements of a suspect class?

The class must be: (1) Determined by unalterable characteristics (immutable); (2) Historically subjected to unequal treatment; and (3) Politically powerless. MNEMONIC: PUNCH (Powerless; UNalterable Characteristics; Historical mistreatment) NOTE: The only two classes that are considered "suspect" are race (or national origin) and alienage. If you read the elements of a suspect class literally, you'd have to wonder why aliens are considered determined by "unalterable characteristics," because aliens can become "non-aliens" by being naturalized. Oh well. That's just the way it is.

After their excellent adventures back in time and to the underworld, Bill and Ted graduate from high school and join the Be Excellent to Each Other Church. They get jobs at a private drug-rehab agency on the condition that they not use any drugs. Bill and Ted ingest peyote while participating in a church service, and get fired from their jobs. The use of peyote is criminal in their state. They apply for unemployment compensation, but are denied benefits on grounds they had been fired for good cause. They challenge their denial of benefits on First Amendment grounds. What result?

The denial is constitutional. The government proscription here burdens religious practices. As a result, if it is a valid and neutral (i.e., not directed at religious practices) proscription of general applicability, it's constitutional even if it incidentally burdens religious practices. In fact, a criminal statute of general applicability not directed at religious practices is simply not subject to challenge on free exercise grounds. The peyote statute is just such a statute. As a result, the fact that receiving unemployment benefits is conditional on not partaking in a religious practice—taking peyote—doesn't make the government action unconstitutional. Employment Division v. Smith, 494 U.S. 872 (1990). RELATED ISSUE: Say, instead, that Bill and Ted were refused unemployment benefits for refusing to work on Sunday, which is something their religion forbids them to do. The result would be different, because when a state conditions receipt of a benefit on conduct prohibited by religious beliefs, the "compelling interest" test still applies—that is, Smith so far only applies where the state is forbidding conduct, not requiring it; therefore, pre-Smith law (in this case, Frazee v. Illinois Dep't of Employment Security, 489 U.S. 829 (1989)) still requires that the state must either (1) grant an exemption from a rule requiring affirmative conduct that happens to burden religion, or (2) show a compelling interest in denying the exemption. Because the state can't show a compelling interest in requiring Sunday work as a condition to unemployment benefits, it must grant the exemption.

Londontown has an ordinance requiring that whenever any organization wants to hold a public parade, it must first obtain a permit from the mayor, who is to make the decision on granting permits depending solely on whether the parade will interfere with times of heavy traffic flow. The Ku Klux Klan applies for a parade permit. Mayor Henry VIII denies the permit when the Klan refuses to allow African Americans and non-Christians to march in the parade. The Klan challenges this refusal in court, claiming a First Amendment violation. What result?

The denial is unconstitutional. The restriction here involves speech in a public forum, so it is valid as a time, place, and manner regulation only if it is content neutral and narrowly tailored to serve a significant governmental interest, and it leaves open alternative media of communication. The problem here is that the mayor's denial of the permit isn't content neutral; the non-discrimination condition is intended to suppress the Klan's free speech. Furthermore, the denial doesn't leave open alternative media of communication, because, under the non-discrimination condition, the Klan can't disseminate its views in public. As a result, the denial is unconstitutional. Invisible Empire of the Knights of the Ku Klux Klan v. Mayor of Thurmont, 700 F. Supp. 281 (Md. 1988). RELATED ISSUE: Granting the license wouldn't constitute impermissible state action in support of discrimination. Public streets of a town are public forums used to communicate ideas and views and granting a permit to do just that doesn't constitute state action.

Amendment IX

The enumeration in the Constitution, of certain rights, shall not be construed to deny or disparage others retained by the people. -- The Ninth Amendment was James Madison's attempt to ensure that the Bill of Rights was not seen as granting to the people of the United States only the specific rights it addressed. In recent years, some have interpreted it as affirming the existence of such "unenumerated" rights outside those expressly protected by the Bill of Rights.

After their TV show goes off the air, Ricky and Lucy Ricardo open up their own nightclub, where Lucy finally gets a chance to be in "the show." She bombs, and so does the club. Lucy's antics start driving Ricky crazy, and he wants to divorce her, but he can't afford the $75 divorce fee charged by the state. He challenges the fee on equal protection grounds. What result?

The fee will be struck down. Although poverty is not a suspect or quasi-suspect classification, when a statute involves a classification based on exercise of a fundamental right, it will be subject to strict scrutiny. The fundamental right of privacy includes marriage rights, which in turn includes the right to dissolve a marriage. The restriction of the right to divorce on the basis of who can pay for it, coupled with the fact that the state holds a monopoly as to the means of dissolving a marriage, means the state is unduly burdening a fundamental right. As a result, the fee is impermissible. Boddie v. Connecticut, 401 U.S. 371 (1971). N.B.: As this hypothetical indicates, watch out for the possibility of heightened scrutiny whenever poverty prevents someone from exercising a fundamental right!

The planet Centauri Prime has a population that is 90% Centauri and 10% human, with a history of discrimination against humans. The Bab 5 school board on Centauri Prime has a policy of laying off Centauri teachers with greater seniority than human teachers if doing so is necessary to maintain racial balance among the faculty. Londo Molari, a Centauri teacher, is laid off before John Sheridan, a human teacher with less seniority. Londo challenges the policy on equal protection grounds. Centauri Prime has a constitution and history of cases identical to ours. What result?

The layoff policy violates equal protection, because it involves a rigid racial quota. The classification here is based on race—layoffs are determined by the teacher's race. Race is a suspect classification, so even though the policy is a form of "affirmative action," it must satisfy strict scrutiny—it must be necessary to promote a compelling interest. While promoting racial diversity in public schools is probably a compelling interest, quota systems like the one here are not considered necessary to promote that goal. (Of course, race could be a factor in determining who gets laid off; it just cannot be the determining factor.) Wygant v. Jackson Bd. of Education, 476 U.S. 267 (1986). RELATED ISSUE: Say it was the federal government that created the layoff policy, not a local government. Under Adarand Construction, Inc. v. Pena, 515 U.S. 200 (1995), strict scrutiny would still have to be used, and the Court would probably decide that this was still a rigid quota system that wasn't a necessary means of combating the effects of past discrimination.

Who were the five justices who voted to invalidate the Guns in School Zones Act in Lopez? Who were the four dissenters?

The majority was composed of Chief Justice Rehnquist and Justices O'Connor, Scalia, Kennedy, and Thomas. The dissenters were Justices Stevens, Souter, Ginsburg, and Breyer. This 5-4 split was common in the Tenth Amendment and Eleventh Amendment cases about federalism.

The Town of Melville has an ordinance that requires any individual seeking to use a sound truck on city streets to first obtain a permit from the chief of police, who has complete discretion in deciding who gets permits. The ordinance is designed to regulate excessive noise. Looey Long, who's running for governor and wants to use a sound truck to proclaim his message to passersby, is denied a permit. He challenges the ordinance on First Amendment grounds. What result?

The ordinance is unconstitutional. It's ostensibly content neutral, and it involves a public forum. Therefore, to constitute a valid time, place, and manner regulation (1) it must be content neutral; (2) it must be narrowly tailored to serve a substantial governmental interest; and (3) there must be alternative channels available for the information. The problem here is the second element; the untrammeled discretion given to the chief of police means the ordinance isn't narrowly drawn. (NOTE: This excessive discretion also constitutes vagueness, which in and of itself makes a regulation invalid.) As a result, the ordinance is invalid. RELATED ISSUE: Say, instead, that the ordinance prohibited sound trucks from emitting loud and raucous noises when operated or standing in public streets, alleys, or thoroughfares of the town. This would be sufficiently narrowly drawn to survive a constitutional challenge.

Privileges or Immunities Clause

The portion of the Fourteenth Amendment that says states may not make laws that abridge the rights granted to people as citizens of the United States.

eminent domain

The power of the government to take private property and convert it into public use. The Fifth Amendment provides that the government may only exercise this power if they provide just compensation to the property owners.

Amendment X

The powers not delegated to the United States by the Constitution, nor prohibited by it to the States, are reserved to the States respectively, or to the people. -- The Tenth Amendment helps to define the concept of federalism, the relationship between Federal and state governments. As Federal activity has increased, so too has the problem of reconciling state and national interests as they apply to the Federal powers to tax, to police, and to regulations such as wage and hour laws, disclosure of personal information in recordkeeping systems, and laws related to strip-mining.

A state regulation denies employment in state-owned transportation facilities to individuals undergoing methadone treatment. The regulation is attacked on equal protection grounds. What level of scrutiny will be used in testing the validity of the regulation?

The rational relation test. As such, the regulation need only be rationally related to a legitimate state interest to be valid. The rational relation test is used for equal protection problems that don't involve a "suspect" class (e.g., race or alienage), don't unduly burden a fundamental right, and don't involve a "quasi-suspect" class (e.g., illegitimacy or gender). Here, the classification involves those in methadone treatment—not a suspect or quasi-suspect class. And public employment is not a fundamental right. Because the state could have any number of valid reasons for the discriminatory classification (i.e., safety), the classifications will be upheld. New York City Transit Authority v. Beazer, 440 U.S. 568 (1979).

The State of Celebration requires that for a political party to get its candidates' names on the ballot, it must submit a nomination petition signed by at least 5% of the number of votes cast in the last election. Otherwise, it will have to conduct a "write-in" campaign. A new political party, the Beer & Pizza Party, challenges the requirement claiming that it unduly burdens its members' freedom of association and therefore is an equal protection violation. What result?

The requirement is valid. As a general rule, when a state action creates a classification that unduly burdens a fundamental right, the action is subject to strict scrutiny. That's the case here; the classification penalizes the exercise of the freedom of association. Under strict scrutiny, the statute is only valid if it is necessary to promote a compelling governmental interest. Although most statutes fail this test, this one doesn't. Preserving the integrity of the electoral process by keeping ballots from being unmanageable or confusing is a compelling interest. Requiring a significant measure of community support to get on the ballot would therefore be valid. Jenness v. Fortson, 403 U.S. 431 (1971).

Amendment IV

The right of the people to be secure in their persons, houses, papers, and effects, against unreasonable searches and seizures, shall not be violated, and no warrants shall issue, but upon probable cause, supported by oath or affirmation, and particularly describing the place to be searched, and the persons or things to be seized. --The Fourth Amendment originally enforced the notion that "each man's home is his castle", secure from unreasonable searches and seizures of property by the government. It protects against arbitrary arrests, and is the basis of the law regarding search warrants, stop-and-frisk, safety inspections, wiretaps, and other forms of surveillance, as well as being central to many other criminal law topics and to privacy law.

STATES AND STANDING

The rules for standing may be relaxed for a state suing on behalf of its citizens. Massachusetts sued the Environmental Protection Agency (EPA), alleging that the agency failed to meet its obligations to regulate greenhouse gases under the federal Clean Air Act. In a 5-4 decision, the Court ruled that the state had met the standing requirements of Article III. The injury requirement was met because of global warming's effect on the state's coastal land. On causation and redressability, the Court rejected the EPA's argument that the effect of regulation on the state's environment would be minimal, finding that any improvement in the condition of the environment, even if minimal, would redress the injury. Massachusetts v. EPA, 127 S. Ct. 1438 (2007).

Harper v. Virginia State Board of Elections

Voting Supreme Court held the use of poll taxes in state elections violates the 14th amendments equal protection clause

City of Cleburne v. Cleburne Living Center

Wealth Classification The Court held the city's refusal to issue a special use permit to built a home for the mentally retarded was unconstitutional because the refusal was based on an irrational prejudice against the mentally disabled.

A State of Cootie statute provides that no retail smoke shop can operate in a wooden building without a license. The state licensing authority consistently refuses to grant such licenses to Cuban Americans, while granting them freely to others. Ricky Ricardo, a Cuban American who's been denied a smoke shop license, challenges the statute on equal protection grounds. What result?

The statute is invalid. Equal protection problems arise when the state government creates a discriminatory classification. The issue here is the intentional nature of the discrimination, as the statute on its face is not discriminatory. However, a statute may be discriminatory in two other ways: It may be unequally administered, or it may have a discriminatory motivation. Here, the statute is administered on a racial basis—Cuban Americans versus non-Cuban Americans. Thus, the statute will be subject to strict scrutiny, and because there's no compelling reason for it, it will fail. Yick Wo v. Hopkins, 118 U.S. 356 (1886). NOTE: Because the statute is not discriminatory on its face, Ricardo would have to prove a discriminatory impact; once he does so, the state would bear the burden of proving some non-discriminatory, constitutionally permissible explanation for the disproportionate impact on Cuban Americans. RELATED ISSUE: Where a statute is discriminatory on its face, the claimant need not show a discriminatory impact.

What standard must a statute meet to withstand intermediate scrutiny?

The statute must be substantially related to an important governmental interest. Intermediate scrutiny is employed where a classification system involves a "quasi-suspect" class, like gender or illegitimacy, and the government intentionally discriminates on the basis of that quasi-suspect class (discriminatory impact alone is insufficient). NOTE: "Substantially related," unlike the "necessary" requirement of strict scrutiny, does not require that the least discriminatory alternative be chosen by the government. NOTE: In the case of gender-based classifications (but not yet illegitimacy), the Court has added another formulation—the gender classification must be supported by "exceedingly persuasive justification." U.S. v. Virginia, 518 U.S. 515 (1996). This seems to require that both the purpose and the means-purpose link be very persuasive and seems to make the intermediate standard a bit tougher to satisfy than previously. For instance, only "actual state purposes," not hypothetical purposes that are not shown to have motivated the state, can count. Id. NOTE: Some classifications involving illegal aliens may also be subject to intermediate scrutiny. The only case indicating this is Plyler v. Doe, 457 U.S. 202 (1982), which gave intermediate scrutiny to rules depriving illegal aliens' children of public education but did not address the review to be given to classifications involving adult aliens. So it's not clear whether other classifications involving illegal aliens would also be subject to intermediate scrutiny.

A State of Bonneville statute prohibits illegitimate children from maintaining wrongful death actions for the deaths of their parents, but allows legitimate children to maintain such actions. Captain Hook's illegitimate son, Fish, wants to file a wrongful death action against Al Ligator for eating his father, but the statute prevents him from doing so. Fish challenges the statute on equal protection grounds. What result?

The statute will be struck down. Equal protection problems arise when the state creates a discriminatory classification. When the classification is based on illegitimacy, as here, the statute is subject to intermediate scrutiny—the law must be substantially related to an important government goal. Here, the state's interest is discouraging people from having illegitimate children. The problem is that the statute punishes the illegitimate children themselves, who cannot control their status. Statutes like this are struck down because of this unfair punishment. Levy v. Louisiana, 391 U.S. 68 (1968).

The legislature of the State of Greenery is very sensitive to environmental issues. Pursuant to this concern, it requires that all infant goods used in the state—including diapers and baby bottles—must be recyclable. Burp 'N Change Baby Co., located in Greenery, makes disposable diapers, as well as baby formula that comes in a non-recyclable plastic six-pack. Burp 'N Change challenges the recycling requirement on equal protection grounds, showing that a neighboring state that has the same ban has not shown any decrease in the number of diapers and bottles thrown away, nor a measurable decrease in total trash. What result?

The statute will be upheld. Equal protection problems arise when a state makes a classification. Here, the classification is manufacturers of recyclables versus manufacturers of non-recyclables. Because this isn't a suspect or quasi-suspect classification, the rational relation test applies—the statute is valid if there is a set of facts imaginable under which the classification would be rationally related to a legitimate state goal. Here, the legislature could feel that banning disposable diapers and bottles would help preserve the environment, a legitimate state goal. It doesn't matter whether the recycling will actually reduce litter, as the legislature needn't prove a statute will have the desired effect. Instead, a law only fails the rational relation test when either it rests on grounds wholly irrelevant to achieving the state's objective, or the objective itself is invalid.

Willie Paul brings an action in federal court for damages against a state highway patrolman who allegedly violated his Fourth Amendment rights by conducting an illegal search of the clothes in his hotel room. The patrolman claims this suit is invalid because of the Eleventh Amendment, which prohibits a citizen from suing a state without the state's permission. What result?

The suit would be allowed (although, of course, Willie Paul may not win it). Note that the Eleventh Amendment only forbids damages actions in federal court against states—it allows federal court actions against individual public officials of a state, if the claim is that the official violated the plaintiff's federal constitutional rights. Here, the patrolman stepped outside his official duty, so the suit will be allowed. Scheuer v. Rhodes, 416 U.S. 232 (1974). NOTE: The Eleventh Amendment also bars suits for injunctions against state officials for violations of state (as opposed to federal) law. Pennhurst State School & Hosp. v. Halderman, 465 U.S. 89 (1984). N.B.: Judges, legislators, and prosecutors enjoy absolute immunity from personal liability in their official actions.

In an effort to appease both gun control advocates and the National Rifle Association, Congress passes the Federal Assault Rifle Safety Act. The Act establishes safety standards for assault rifles manufactured domestically. One provision of the Act creates a three-member "Assault Rifle Advisory Team" to investigate the safety standards and prosecute those who violate the Act. The Act provides that the President will appoint the chairman of the team, the President pro tem of the Senate will appoint one member, and the Speaker of the House will appoint the third member. What's the most significant constitutional problem with these provisions?

The team does not have the power to enforce the Act, because some of the members are appointed by the legislature. Under the Constitution, Congress has the power to make laws, and the President has the power, in essence, to enforce laws. The problem here is that the team has the power to prosecute, which is an enforcement power exclusively within the province of the President. What Congress is doing here is delegating to the team the power to prosecute, which is a power Congress doesn't have (and thus cannot delegate). As such, all members of the team would have to be appointed by the President and approved by the Senate, under Article II, §2 of the Constitution. Buckley v. Valeo, 424 U.S. 1 (1976). RELATED ISSUE: Faced with this problem, a court would most likely allow the team to continue functioning in every respect except prosecuting violations of the Act. This judicial interpretation would validate the team's function to the extent possible. Because Congress can delegate its legislative power, this interpretation would tailor the team to the confines of the Constitution. NOTE: Congress' power to delegate allows it to delegate legislative functions to the President, while the converse isn't true—the President couldn't delegate enforcement functions to Congress.

Congress becomes concerned with the welfare of a migrating bird, the yellow-breasted wood titmouse. It passes legislation that bars the killing of yellow-breasted wood titmice in the United States. The statute is struck down as being beyond congressional power. Thereafter, the United States and Canada enter a treaty concerning migratory birds that, among other things, prevents the killing of yellow-breasted wood titmice in the United States. The treaty is properly ratified by the Senate. The State of Yakulta brings suit in federal court, claiming the treaty (as applied to birds found in Yakulta) is an improper invasion of rights reserved to the states under the Tenth Amendment. What result?

The treaty is valid. Missouri v. Holland, 252 U.S. 416 (1920). The issue here is whether treaties can address matters beyond Congress' enumerated and implied powers. The rule is that, as long as the treaty is otherwise valid, it can address matters otherwise outside the scope of congressional control, and it will be binding on the states, as with any other federal law, under the Supremacy Clause. A treaty is valid if: The President ratifies it with consent from two-thirds of Senators; It deals with matters that are properly subject to negotiation with a foreign country (which is almost always true, because foreign countries normally only enter into treaties that concern them); and The treaty doesn't violate any constitutional guarantees or prohibitions (like Due Process). Here, the protection of birds that migrate between the United States and Canada is a national problem concerning both nations, so the Tenth Amendment wouldn't provide a means for the state to avoid application of the treaty.

Washington v. Glucksberg

This 1997 Supreme Court case held that the right to privacy protected by the 14th Amendment Due Process Clause does NOT include the right to assisted suicide ("euthanasia"). Three states currently allow assisted suicide. It is a serious crime everywhere else.

Marbury v. Madison

This case establishes the Supreme Court's power of Judicial Review

Debs v. United States

This was a Supreme Court decision that upheld the Espionage Act of 1917. He, leader of the Socialist Party, fought the draft during World War I. In Canton, Ohio, he made a speech that protested U.S. involvement in the war. After the speech, he was arrested under the Espionage Act of 1917. After being convicted, he was sentenced to ten years in prison.

What is the test to determine whether a text or work is obscene and thus not protected by the First Amendment?

To be obscene, all three of the following requirements must be met: (1) The average person (which may include sensitive adults, but may not include children), applying contemporary community standards, would find that the text or work, taken as a whole, appeals to the "prurient interest"; (2) The text or work depicts or describes, in a "patently offensive way," sexual conduct specifically defined by the applicable state law; and (3) The text or work, taken as a whole, lacks serious literary, artistic, political, or scientific value. These requirements are collectively known as the Miller test, after Miller v. California, 413 U.S. 15 (1973).

What are the President's powers in the area of foreign affairs?

To: (1) Recognize (and withdraw recognition of) foreign governments; (2) Receive ambassadors and other public ministers; and (3) Act on the nation's behalf in day-to-day dealings with foreign governments. NOTE: These are exclusive powers. The President shares with Congress the power to determine foreign policy (although the President has a distinct advantage, in that he controls information and can respond quickly to developments abroad). Remember that the federal government enjoys exclusive control over foreign affairs; the states have none.

Saenz v. Roe

Travel The Court held the state law that limited the amount of welfare benefits averrable to newly arrived residents was unconstitutional under the privileges or immunities clause because the right to travel/relocate is protected by the clause.

When does state action become an issue?

When a plaintiff alleges that his constitutional rights have been violated by a private person or entity. That's because nearly all protections of individual rights apply only to governmental entities; the only constitutional provision that addresses private (as opposed to governmental) conduct is the Thirteenth Amendment, which involves slavery. Thus, whenever a suit brought against a private individual addresses anything other than slavery, the question is whether the defendant's actions constitute "state" action as proscribed by constitutional provisions. (Note that "state action" refers to the act of any government, be it federal, state, or local, not just a state government.) NOTE: In a procedural sense, a defendant raises the state action issue by claiming that it wasn't possible for him to violate the constitutional provision in question (typically due process, equal protection, or the First Amendment), because he isn't part of the government. If the plaintiff proves the defendant has deprived him of a constitutionally guaranteed right, and the Court determines that state action was involved, the Court will fashion an appropriate remedy and order an end to the defendant's unconstitutional conduct.

You already know that the rational relation test is very difficult to fail, and that if a statute isn't subject to strict or intermediate scrutiny, this is probably the kiss of death for an equal protection claim. However, there are a few situations in which laws subject to the rational relation test are struck down. When is this most likely to happen?

When either the law rests on grounds wholly irrelevant to achieving the state's objective or the objective itself is invalid. For instance, say a state charges a higher tax on out-of-state businesses than on in-state businesses. If the state's motive is to protect local businesses from competition, the interest is invalid, so the tax will fail the rational relation test. Metropolitan Life v. Ward, 470 U.S. 869 (1985). Or, let's say the state amends its constitution to prohibit the state, or any city in it, from giving gay individuals protection from job discrimination. The state's "bare desire to harm a politically unpopular group" (such as gay individuals) can't be a legitimate governmental interest. So the amendment must be struck down, even though it's only subject to the rational relation test. Romer v. Evans, 517 U.S. 620 (1996). RELATED ISSUE: The Court relied on substantive due process to invalidate a Texas statute that penalized homosexual, but not heterosexual, sodomy, concluding that the right to privacy protected by the Due Process Clause of the Fourteenth Amendment included a right to adult consensual homosexual activity. Justice Sandra Day O'Connor's concurrence argued that the Texas statute should have been invalidated under the Equal Protection Clause because it singled out homosexuals for government disfavor, failing rational basis scrutiny. The Court relied on both substantive due process and equal protection in ruling for same-sex marriage. Obergefell v. Hodges, 135 S. Ct. 2584 (2015).

In the context of classifications based on gender, what governmental interests are considered "important"?

When governments defend their gender-based classifications against equal protection attack, most of the interests they assert have been found to be "important." Two common governmental interests that are found to be "important" are (1) remedying past discrimination against women, and (2) preventing illegitimate teen pregnancies. NOTE: Two governmental interests that do not meet the importance requirement are (1) administrative convenience, and (2) providing women a choice of coed educational environments while denying that option to men.

ANALYZING SUBSTANTIVE DUE PROCESS PROBLEMS

When legislation impairs a right, ask: Is the right a "fundamental personal" right? If yes, go to #1; if no, go to #2. Yes, the right is a "fundamental personal" right. These are the First Amendment rights (e.g., speech, press, religion, assembly, petition), interstate travel, voting, privacy (e.g., marriage, contraception, procreation, raising children, family interest), death, and fairness in the criminal process (e.g., right to counsel on first appeal). Here, the legislation must meet the "compelling state interest" test—it must be necessary to promote a compelling governmental interest. No, the right is not a "fundamental personal" right. (Instead, the statute is merely regulating social or economic interests of lesser importance.) Here, the statute is only subject to the "rational relation" test—if there is a set of facts imaginable that would make the law a reasonable means to achieve a legitimate governmental purpose, the law is valid. Such laws include public health and safety measures and all kinds of business regulations, including trade practices, wage and hour regulations, price controls, and bans on discrimination against union (or non-union) personnel.

What is the "market participant" rule relating to interstate commerce?

When the state is a market participant (i.e., buyer or seller of goods), it may discriminate in favor of local businesses—that is, it could buy from local businesses exclusively (or preferentially); similarly, it could sell to them exclusively/preferentially. RATIONALE: The Commerce Clause only prohibits discriminatory regulation; when a state itself buys or sells, it's a participant, not a regulator. EXAMPLES: A state could require that all cement produced by a state-owned factory be sold only to state residents; it could sell state-owned resources to state residents at a lower price than to out-of-staters; it could offer a company a cash bonus or tax exemption for opening a factory in state. It could not, however, require that a privately owned factory sell its goods only to people in state; it couldn't pass laws requiring a company to give employment preferences to state residents. N.B.: Keep in mind that the fact a state is a market participant doesn't insulate it entirely from violating the Commerce Clause. Situations in which the state bears the cost of providing economic benefits are permissible, because the state is spending its own tax revenues, as it has a right to do. However, where what the state is doing is really shifting the cost of the local benefit to out-of-staters (e.g., by requiring privately owned local businesses to sell to people in-state at a discount), the action is a kind of regulation, and it violates the Commerce Clause. N.B.: The state as a market participant would still be subject to the interstate Privileges & Immunities Clause, which limits its leeway as a market participant. Watch for this whenever a state or city interferes with private sector employment (e.g., by requiring a contractor on a state building project to hire only state residents for the project; this could violate the interstate P & I Clause).

Tired of competing with Japanese car companies, car makers in Detroit persuade the State of Michigan to declare war on Japan. The state militia loads Lincoln Continentals into portable rocket launchers and aims them at Tokyo. The federal government has not declared war on Japan and has no intention of doing so. Is Michigan's declaration of war "preempted"?

Yes, because states have no power to declare war. Preemption is an issue whenever the federal government and a state regulate the same matter, or a state regulates something it has no authority to regulate. When the Constitution prohibits state action in an area, any state regulation in that field is void. Such areas include coining money, foreign affairs, and—as here—declaring war. Because Michigan is constitutionally barred from declaring war, its doing so is preempted.

The Federal Motor Vehicle Safety Act provides, in part, that car companies may use seat belts to meet federal occupant crash protection standards. Furthermore, the Act provides that "no state shall have any authority to establish, or to continue in effect, any motor vehicle safety standard that is not identical to the federal standard" applicable to the same aspect of vehicle performance. Quasimodo is disfigured in a car accident. He brings a product liability suit in state court against Field Marshal Motors, the manufacturer of his car, alleging a design defect in that his car lacked air bags. Under state common law the lack of air bags could constitute a design defect. Does the federal law preempt the state law?

Yes, because the federal law expressly covers the entire area of crash protection. Here, the state common law would require safety measures in addition to the federal requirements. The federal law is satisfied by seat belts; the state common law might require air bags. While there's no direct conflict, the state law will nonetheless be invalid if Congress intended to preempt the entire field. That's the case here, because the federal law expressly requires that state law mirror the federal standard applicable to the same aspect of vehicle performance. As a result, the federal law will preempt the state law. Cox v. Baltimore County, 646 F. Supp. 761 (D. Md. 1986); Wood v. General Motors, 865 F.2d 395 (1st Cir. 1988); Taylor v. General Motors, 875 F.2d 816 (11th Cir. 1989).

Can words, unaccompanied by pictures, ever be considered obscene?

Yes, because words alone can meet the three-pronged obscenity test (overall appeal to prurient interest; patently offensive under prevailing community standard relating to sex; and lacking in "serious" literary, artistic, political, or scientific value). This is known as the Miller test. RELATED ISSUE: A song can be obscene, as its lyrics can fit the obscenity test. But the music can't be obscene unless the entire work, music plus lyrics, lacks serious artistic value; so obscene lyrics might be saved by a serious musical component. Luke Records v. Navarro, 960 F.2d 134 (11th Cir. 1992).

Miss Brooks is a fourth-grade teacher. Her class is a group of little terrors. The school board decides that henceforth, Miss Brooks' class, and every other fourth-grade class, shall have the Ten Commandments posted on each classroom's blackboard. (The school board figures that the fourth graders will be afraid to violate the word of God and will thus be kept in line.) Is this posting order a violation of the Establishment Clause?

Yes, probably. Under Lemon v. Kurtzman, 403 U.S. 602 (1971), a governmental action that arguably furthers religion must have a secular purpose, must not have the primary effect of advancing religion, and must not foster excessive government entanglement with religion. The posting requirement seems to have a secular purpose (to keep the kids in line), so the fact that it also may have a religious purpose is not fatal. However, the Court has also been vigilant in invalidating laws based on a "sham secular purpose," so it would examine the purpose carefully. A court would also probably hold that the requirement has the primary effect of advancing religion, thus making it an Establishment Clause violation. (If the court held that the primary effect of the posting rule was to maintain discipline rather than to advance religion, the rule might survive—it's probably not an excessive entanglement.) NOTE: The reading of a "prayer" in school is clearly a violation of the Establishment Clause. Engel v. Vitale, 370 U.S. 421 (1962).

Are affirmative action programs subject to strict scrutiny?

Yes, regardless of whether the program is a federal program or a state/local program. See Adarand Constructors, Inc. v. Pena, 515 U.S. 200 (1995) (holding that congressionally authorized programs must satisfy strict scrutiny). Even though such programs are strictly scrutinized, they may be upheld if they meet the following requirements: (1) They must be narrowly tailored (i.e., not overbroad; they should be based on pertinent racial percentages in the relevant population); (2) They must not involve a rigid racial quota, but rather use race as a "plus"; and (3) There must be an adequate showing of specific past discrimination by the entity doing the affirmative action (e.g., city) in question; a mere history of national racism or a statistical demonstration won't suffice); or they must enhance diversity in a way that a colorblind program would not. Richmond v. J. A. Croson, 488 U.S. 469 (1989). NOTE: Where state and local governments are concerned, remedying an entity's own past improper discrimination is a compelling interest; remedying the general effects of society's discrimination is not a compelling interest. RELATED ISSUE: Enhancing diversity is a compelling state interest that allows race to be considered as a factor in college and university admissions programs. Strict racial quotas are impermissible, University of California Regents v. Bakke, 438 U.S. 265 (1978), as are mechanical, non-"holistic" approaches resembling quotas, Grutter v. Bollinger, 539 U.S. 306 (2003) and Gratz v. Bollinger, 539 U.S. 244 (2003).

Is the President the only source of constitutional "executive power"?

Yes, under Article II, §1. However, many executive responsibilities are delegated to others within the executive branch. SIGNIFICANCE: The executive branch has exclusive law enforcement powers—that is, the executive carries out the laws Congress makes. Congress could not, for instance, delegate the power to conduct litigation to a committee of its own. NOTE: Unlike most congressional powers, many executive powers are implied—so they needn't fall within an enumerated power found in the Constitution. NOTE: Most issues involving presidential power concern the separation of powers and are discussed in that topic under the code SOP.

In response to the Second World War, Congress passed the Non-Detention Act, which states that "no citizen shall be imprisoned or otherwise detained by the United States except pursuant to an Act of Congress." In response to the terrorist attacks of September 11, 2001, Congress passed the Authorization for Use of Military Force (AUMF). Hamdi, a U.S. citizen, was captured in Afghanistan and detained by the executive branch as an "enemy combatant." Does the President have the authority to detain Hamdi?

Yes. A plurality of the Court in Hamdi v. Rumsfeld, 542 U.S. 507 (2004), ruled that the AUMF provided congressional authorization for the President's detention of Hamdi, and that the President had not violated the Non-Detention Act. NOTE: Justice Thomas, the fifth vote to uphold the president's power, wrote that the President had inherent power as Commander in Chief to detain U.S. citizens as enemy combatants. NOTE: Justice Scalia, joined by Justice Stevens, concluded that a U.S. citizen cannot be held by the government without charges or trial unless Congress suspends the writ of habeas corpus. NOTE: The Court also ruled that Hamdi must be accorded some form of due process. Hamdi v. Rumsfeld, 542 U.S. 507 (2004).

State Prisoner Tony is paraplegic and uses a wheelchair. State prison authorities confine him to a small cell that is so narrow that he cannot move his wheelchair around. They also refuse his requests for trips to the restroom so that he frequently has to sit in his own waste. Can Tony sue the State for violations of the Americans with Disabilities Act?

Yes. Although in earlier cases the Court had held that Congress could not use its §5 of the Fourteenth Amendment powers to abrogate state sovereign immunity in the Americans with Disabilities Act (ADA), the Court held unanimously in U.S. v. Georgia, 546 U.S. 151 (2006), that a paraplegic prisoner could sue the state under the ADA in these circumstances. The reasoning was that the prison's conduct violated the Eighth Amendment's ban on cruel and unusual punishment, which is applied to the states through the Fourteenth Amendment. Because treating a prisoner with cruel and unusual punishment is a constitutional violation under the Fourteenth Amendment, Congress could validly abrogate the states' immunity.

Relations have been strained between the United States and the country of Iguana ever since a former U.S. President insulted Iguanians by appointing a former child movie star as ambassador to Iguana. Iguana has, for some time, severely restricted the emigration of Iguanians to the United States. To ease the tension, the President, with the advice and consent of the Senate, enters into a treaty with Iguana, permitting the citizens of each country unrestricted travel between the two. This causes a mass exodus from Iguana to the United States. In response, Iguana clamps down and forbids an Iguanian family from emigrating to the United States. The President terminates the treaty because of this breach, without consulting the Senate. Is the termination valid?

Yes. Although treaties may be created only with the advice and consent of the Senate, the President may, himself, continue or terminate a treaty without the Senate's consent. In fact, the President's termination of a treaty would be considered a non-justiciable political question, thus removing any judicial roadblocks from a President's power to nullify treaties. Goldwater v. Carter, 444 U.S. 996 (1979).

Does Congress have the authority to create inferior or lower federal courts?

Yes. Article III, §1 of the Constitution says that the "Judicial Power of the United States, shall be vested . . . in such inferior Courts as the Congress may from time to time ordain and establish."

Could a state statute discriminating against men violate equal protection?

Yes. As with statutes discriminating against women, statutes discriminating against men will be found to violate equal protection unless the law substantially relates to an important government objective (the "intermediate scrutiny" test). However, keep in mind that the state has an important interest in compensating women for past discrimination and in preventing illegitimate teen pregnancies. EXAMPLES: A state law provides that only husbands and not wives may be ordered to pay alimony. The law is invalid, because the aim is to help all spouses in need—thus, it should apply to men and women alike. Orr v. Orr, 440 U.S. 268 (1979). However, the state may define statutory rape so that only men may commit it, as there is an important state interest in preventing teen pregnancies. Michael M. v. Superior Court, 450 U.S. 464 (1981).

Can Congress forbid states from taxing interstate commerce?

Yes. Congress can, under its commerce power, forbid states from passing taxes that affect interstate commerce. Prudential Ins. Co. v. Benjamin, 328 U.S. 408 (1946). NOTE: If there isn't any federal legislation, the states are free to place a fair level of tax on interstate commerce, because such commerce must be expected to shoulder its fair share of a state's expenses. NOTE: State taxes that discriminate against out-of-state residents also violate the Article IV, §2, interstate Privileges & Immunities Clause. NOTE: State taxes that discriminate against interstate commerce violate the Equal Protection Clause if they are not rationally related to a legitimate state purpose (e.g., the state cannot deny a tax exemption to a corporation solely due to its being incorporated in another state). WHYY v. Borough of Glassboro, 393 U.S. 117 (1968).

Congress enacts a statute that removes the Supreme Court's appellate jurisdiction for cases involving abortion. Is the statute valid?

Yes. Congress is expressly granted the power to regulate the appellate (but not original) jurisdiction of federal courts in Article III, §2 of the Constitution. Ex parte McCardle, 74 U.S. 506 (1869). Although it's not clear how broad this power is, it's not unlimited; therefore, it's possible that even if Congress eliminated Supreme Court review of certain types of cases (like abortion cases, here), it would have to permit some lower federal court to retain jurisdiction over such cases. U.S. v. Klein, 80 U.S. 128 (1872). RELATED ISSUE: Congress can't change the Supreme Court's original jurisdiction.

Bard's Bookstores did a lot of business with Shakespeare State University, whose students always need books. Unfortunately Bard's was not so good at business and went bankrupt. Mr. Cats was appointed supervisor of the bankrupt estate. He sued Shakespeare in Bankruptcy Court, alleging that Bard's had illegally transferred money to Shakespeare before it entered bankruptcy. Shakespeare argued that the case should be dismissed because of state sovereign immunity. Should the Bankruptcy Court take jurisdiction of the lawsuit?

Yes. In Central Virginia Community College v. Katz, 546 U.S. 356 (2006), a 5-4 decision by Justice Stevens, the Court ruled that Congress has the power to abrogate state sovereign immunity under the Bankruptcy Clause of Article I, §8, cl. 4, which provides that Congress shall have the power to establish "uniform Laws on the subject of Bankruptcies throughout the United States." NOTE: The Bankruptcy Clause is the only Article I power that the Court has allowed to abrogate state sovereign immunity. NOTE: Congress may abrogate state sovereign immunity using its powers under §5 of the Fourteenth Amendment.

Did Congress have the authority to charter the Second Bank of the United States?

Yes. In McCulloch v. Maryland, 17 U.S. 316 (1819), one of the most important early Court decisions, Chief Justice Marshall explained that the power to create the bank could be implied from the Constitution's text. Although the power to create a bank was not explicitly stated in the Constitution, Marshall relied on the "Necessary and Proper" Clause as a justification for the bank's creation. According to Marshall, "necessary" does not mean "absolutely necessary." As long as the means is rationally related to a constitutionally specified object, the means is constitutional. The bank was rationally related to Congress' enumerated powers to collect taxes, to borrow money, and to regulate commerce.

Des Cartes leads a group of devil worshippers. Part of their ritual is to dissect a live goat, which results in excruciating pain to the animal. Des is convicted under a state statute of general applicability prohibiting cruelty to animals. He appeals his conviction. Will his conviction stand?

Yes. It's true that the statute here burdens Des' religious practices. But, the rule is that when a statute is a valid and neutral (i.e., not directed at religious practices) law of general applicability, it's constitutional even if it incidentally burdens religious practices. Here, the statute is a valid exercise of the police power and is religion neutral. As such, Des' conviction would likely stand. Employment Division v. Smith, 494 U.S. 872 (1990). N.B.: If there were exceptions in the statute for other, non-religious, painful dissection of animals, Des would probably be able to prove that the law unfairly discriminated against his free exercise of religion. Church of the Lukumi Babalu Aye v. Hialeah, 508 U.S. 520 (1993).

Are there any circumstances under which states can regulate interstate commerce?

Yes. Non-discriminatory regulation is allowed as long as it meets this balancing test—the burden on interstate commerce (e.g., cost and difficulty of compliance, inefficiency created) is weighed against the strength of the state interest in the regulation to determine whether the burden on interstate commerce is "clearly excessive." Pike v. Bruce Church, Inc., 397 U.S. 137 (1970). In general, there is a strong presumption of constitutionality of non-discriminatory statutes where the state interest is safety, health, or social welfare; courts are much less likely to uphold statutes that only protect local economic interests. NOTE: Congress does, in practice, enjoy exclusive control over foreign commerce—and foreign commerce is interpreted broadly (e.g., a shipment from San Francisco to San Diego could be considered "foreign commerce" if the shipment was on the "high seas"). Lord v. Steamship Co., 102 U.S. 541 (1880).

Congress passes a statute making it a crime to create, sell, or possess any depiction of animal cruelty in violation of federal or state law existing wherever the video was created, sold, or possessed. The intention of the statute is to ban so-called crush videos depicting women crushing small animals in high heels. Don Mexico is charged with selling videos of dogfights, and he challenges the statute's infringement on his First Amendment right to free speech. Will he prevail?

Yes. On similar facts the Court recently held in U.S. v. Stevens, 130 S. Ct. 1577 (2010), that the animal-crush video ban is unconstitutional as overbroad. Animal rights activists argued that the government had an interest in preventing the harm to animals that inevitably comes as a result of the filming of the videos, similar to the state interest in child pornography bans. However, the Court ruled in favor of the defendant because the statute criminalized a range of content too great to be consistent with the First Amendment, as it could apply to situations in which the content of the video was legal where it was created, but not where it was possessed or sold. NOTE: The Court did note that there could be certain portrayals of cruelty to animals that the government could ban to prevent harm to animals. The Court withheld deciding whether a statute that criminalized only crush videos would be constitutional.

Old Mother Hubbard receives a notice in the mail informing her that her welfare benefits have just been cut off. This is the first that Mother has heard about the possibility of losing her benefits. Mother claims that this action violates her due process rights. Does it?

Yes. Procedural due process rights only attach when one is deprived of a property or liberty right. Deprivation of one's welfare benefits is a right that qualifies for protection. Determining the amount of protection necessary requires balancing the importance of the property or liberty interest in question and the risk an erroneous deprivation in a particular procedure would create (considering the probable value of any additional safeguards) against the importance to the government of the function in question and the administrative and fiscal expense of a particular safeguard. Mathews v. Eldridge, 424 U.S. 319 (1976). The Court has held that a welfare recipient is entitled to notice and an evidentiary hearing prior to the termination of such benefits. Goldberg v. Kelly, 397 U.S. 254 (1970).

Would a person's belief in secular humanism as a religion be protected by the First Amendment?

Yes. Religious beliefs must be held in good faith, but they need not be theistic to be protected by the First Amendment.

Congress enacted the comprehensive Guns Bill aimed at regulating the flow of guns in interstate commerce. The Bill authorizes the creation of a national computer registry containing background information on potential gun purchasers. Because the national registry will take some time to develop, however, the legislation orders local law enforcement officials to conduct background checks on all gun purchasers in their states until the national registry is ready. Sheriff objects to collecting the information and challenges the constitutionality of the requirement. Will Sheriff win his lawsuit?

Yes. The Court held in Printz v. U.S., 521 U.S. 898 (1997) that the anti-commandeering rule in New York extended to the executive branch. The Congress may not order state executive branch officials to enforce federal regulations. NOTE: In Printz, Justice Scalia wrote that the Act violated separation of powers principles because it delegated federal executive power to state and local law enforcement.

Art Beats suffered a heart attack while running a marathon. At the hospital a balloon catheter, which is a medical device manufactured by Medtrials, Inc., was inserted into Art's heart by a surgeon. Later the balloon catheter popped and Art died. His family sued in state court under state tort law arguing that the balloon was defectively designed and labeled. The federal Medical Devices Act (MDA) provides that once a medical device receives premarket approval from the Food and Drug Administration (FDA), "no state or local government may impose any requirement that either relates to the safety or effectiveness of the device, or that is different from, or in addition to, any FDA requirement applicable to the device." Medtrials' balloon catheter received premarket approval from the FDA. Is Art's lawsuit preempted?

Yes. The Court ruled 8-1 in Riegel v. Medtronic, 128 S. Ct. 999 (2008), that a similar lawsuit was expressly preempted by the MDA. NOTE: Express preemption is present when the law specifically or expressly says that state or local law is preempted. Contrast express with implied preemption, where Congress' action suggests preemption. The two main types of implied preemption are field preemption and conflicts preemption.

Congress wants to ensure that the money it gives to state and federal governments is used for appropriate projects and not wasted or used for graft. It passes legislation, the Anti-Bribery Act (ABA), which imposes federal criminal penalties on anyone who bribes state or local officials who work for entities that receive at least $10,000 in federal money. Crooked Builder wants to build a new housing development in State of Corruption and gives State Corrupt Official $5,000 to help him get approval for his building permits. Builder is convicted under the federal statute and appeals its constitutionality. Is the ABA constitutional?

Yes. The Court unanimously upheld a statute proscribing bribery of state, local, and tribal officials of entities that receive at least $10,000 in federal funds as a valid use of Congress' spending powers. The Court also stated that the Act was necessary and proper to protect the integrity of the use of federal funds. Sabri v. U.S., 541 U.S. 600 (2004).

After Movie Star was stalked and killed by a man who got her home address from State's Department of Motor Vehicles, Congress passed the Driver's Privacy Protection Act (DPPA). The DPPA protects the privacy of drivers by prohibiting state motor vehicle departments from selling in interstate commerce drivers' personal information, including their Social Security numbers, home addresses, phone numbers, photographs, and medical information. State of Profit challenges the law, arguing that the law exceeds Congress' commerce powers and that it commandeers the state government in violation of New York and Printz. Is the DPPA constitutional?

Yes. The Court unanimously upheld the DPPA in Reno v. Condon, 528 U.S. 141 (2000). First, the law fell within Congress' Commerce Clause powers because the information sold by the states was used by marketers, manufacturers, and others engaged in interstate commerce to contact consumers with customized solicitations. Therefore the drivers' information was an article of commerce. Second, the Court held that the Act did not commandeer the states in their regulatory or enforcement capacity but instead regulated them as owners of databases, just as it regulated private owners of databases. On these facts, the State of Profit is an owner of information, not a sovereign, and it has not been commandeered.

The Town of Weinerschnitzel is a tiny resort. Its one attraction is its annual five-month-long Oktoberfest, where tourists flock to the town to enjoy drinking beer, wearing long stockings, drinking beer, folk dancing, and drinking beer. Most of the town's payroll consists of Oktoberfest employees. They are paid the federal minimum wage; however, during the Oktoberfest season, when they work seven ten-hour days every week, they are not paid anything in addition to their regular wage. Under the federal Fair Labor Standards Act (FLSA), they would have to be paid time-and-a-half for overtime. The federal government demands that Weinerschnitzel comply with the overtime pay provision. Weinerschnitzel calculates that, if it does so, it may have to bankrupt itself, and it challenges the statute on Tenth Amendment grounds. Will a court enforce the overtime provision against Weinerschnitzel?

Yes. The FLSA is an exercise of Congress' commerce power. The issue here is whether Congress' regulation of the workers' salaries violates the Tenth Amendment. Because regulating pay and working hours is a valid exercise of the commerce power, and because the Tenth Amendment provides little practical limitation to the scope of the commerce power, Weinerschnitzel will have to comply with the provision regardless of the consequences. Garcia v. San Antonio, 469 U.S. 528 (1985). RELATED ISSUE: In Alden v. Maine, the Court held that sovereign immunity bars suits against states in state and federal courts to enforce the FLSA. (See FJ/11A 6.)

The State of Mission Law School accepts 350 of its 3,500 applicants. One of its goals in admitting those students is to build a student body of diverse backgrounds and interests. Mission Law School has a particular commitment to racial diversity, and works to include students from groups that have faced historical discrimination, namely African Americans, Hispanics, and Native Americans. The school's goal is to have a critical mass of unrepresented minority students. To promote this goal of diversity, the Law School's admissions staff put together a system in which any applicant is admitted if he or she receives 100 points. The staff adds an extra 20 points to the files of all non-white minority applicants. Bobbie Bratz, a white student with a higher grade point average and LSAT score than some of the accepted minority students, challenges the program as a violation of equal protection. Will her challenge succeed?

Yes. The Law School program does not survive strict scrutiny. All racial classifications are subject to strict scrutiny, which means that they are constitutional only if they are narrowly tailored to further compelling governmental interests. In Grutter v. Bollinger, 539 U.S. 306 (2003), the Court recognized diversity as a compelling government interest that could justify the use of racial classifications. In Gratz v. Bollinger, 539 U.S. 244 (2003), however, an undergraduate program similar to the hypothetical because it awarded 20 points for racial classifications was invalidated because there was no individualized selection process and no meaningful individualized review.

Larry Loser has just completed a federal prison sentence for child molestation and is looking forward to his freedom. Then he learns that the federal government plans to civilly commit him under its Sexual Danger Statute. The statute lets the government civilly commit sexual offenders who are mentally ill and dangerous to others. Larry knows that the federal government is one of enumerated powers and that Congress cannot act unless it can point to the constitutional source of its powers. Is there any power of Congress that authorizes the Sexual Danger Statute?

Yes. The Necessary and Proper Clause, Article I, §8, cl. 18. In U.S. v. Comstock, 130 S. Ct. 1949 (2010), Justice Breyer relied on the Necessary and Proper Clause to uphold a civil commitment statute for sexual offenders. The majority rejected Larry's argument "that, when legislating pursuant to the Necessary and Proper Clause, Congress' authority can be no more than one step removed from a specifically enumerated power." Because Congress already has broad power to criminalize conduct and protect people against dangerous criminals, the Sexual Danger Act had a "means-end rationality" connection to Congress' powers. NOTE: Justice Thomas' dissent warned that the Court had breathed new life into the Necessary and Proper Clause and almost transformed it into a federal police power.

Does the President enjoy absolute immunity from civil damages in carrying out his official duties?

Yes. The President has absolute immunity from civil liability for his official acts. This extends to the "outer perimeter" of his authority under the Constitution. Nixon v. Fitzgerald, 457 U.S. 731 (1982). NOTE: But the President does not have immunity—even "qualified" immunity—for acts that are completely unrelated to the carrying out of his job. See Clinton v. Jones, 520 U.S. 681 (1997) (no immunity for acts taken before President took office, such as the sexual harassment claimed here). NOTE: Presidential aides generally have only a qualified immunity, such that they can be subject to civil damages if their actions violate statutory or constitutional rights, which are clearly established, and of which a reasonable person would know. Mitchell v. Forsyth, 472 U.S. 511 (1985).

Acme Flange Co. operates Dullsville, a "company town"—that is, Acme supplies all traditional municipal functions (e.g., police/fire protection, local schools, etc.) for the residents of Dullsville. Even though Dullsville is privately owned, will it be treated as a regular town for purposes of the Fourteenth Amendment?

Yes. The actions of a private entity only rise to the level of state action when the state either provided a "mantle of authority" for the actions, or was significantly involved in, or encouraged, those actions. One way for this to happen occurs when the state delegates to a private actor powers "exclusively reserved to the State." The operation of a "company town"—in which the state delegates to a private entity the authority to provide all the services that towns usually provide—is one of the few types of delegation that is so extensive as to qualify. Company towns are therefore regular towns for purposes of the Fourteenth Amendment; thus, they cannot, for instance, prohibit the dissemination of literature on their streets. Marsh v. Alabama, 326 U.S. 501 (1946). COMPARE: A privately owned shopping mall does not exercise exclusive governmental functions, and thus, does not have to obey the Fourteenth Amendment (e.g., malls can prohibit picketing on the premises). Hudgens v. NLRB, 424 U.S. 507 (1976).

A state law provides that veterans will be given preference in public employment opportunities. This results in far greater public employment for men than for women. Will the law be upheld when challenged on equal protection grounds?

Yes. The discrimination here is de facto—the law was not intended to discriminate against women, it simply has a disproportionate impact on women. Because the discrimination is not intentional, the law will not receive "intermediate" scrutiny (requiring that the law be substantially related to important government interests), as is usual with gender discrimination. Instead, it will only have to meet the rational relation test, and need only be rationally related to a legitimate state interest. Under this test, the law will be upheld. Personnel Administrator v. Feeney, 442 U.S. 256 (1979).

The State of Illiana sets up a private school tuition voucher program. Under the program, any elementary or middle school student who lives in what the statute defines as a "failing public school district" may receive $2,000 per year of tuition assistance toward any private school that accepts the student. The statute that sets up this program also includes other provisions supplying special aid, worth about the same $2,000 per student, for any student in a failing district who wishes to remain in public school but receive after-school tutoring at the public school. About 10% of eligible students choose to participate in the private school voucher program; of these, about 97% attend parochial school (mainly because non-parochial private schools in the state are so expensive that the $2,000 of assistance doesn't defray enough of the tuition bill to make much of a difference). Is the voucher program constitutional?

Yes. The facts here are very similar, although not identical, to the facts in Zelman v. Simmons-Harris, 536 U.S. 639 (2002). There, the Court by a 5-4 vote found a Cleveland tuition voucher program not to be a violation of the Establishment Clause. The majority's main point was that any aid to religion came about by means of the individual "private choices" of each student and parent, not by means of the direct decision making of the state. NOTE: In Zelman, the fact that the state also tried to give special assistance to those remaining in the public schools seemed to make some difference to the majority, but it's not clear how much. So it's possible that without the after-school public school tutoring program specified here, the voucher program would violate the Constitution—but this seems unlikely.

Under the federal Age Discrimination in Employment Act (ADEA), Congress has specified that no employer, public or private, may discriminate on the basis of age against an employee, no matter how old. The ADEA by its terms applies to all employers, public or private. The Act includes a provision allowing the person discriminated against to recover money damages in "any state or federal court of competent jurisdiction." Methuselah, who is 80, worked as an actuary for the State of Efficiency until last year, when he was fired solely on account of what his boss called "an age so advanced that no one that old could possibly be a competent worker." Meth has sued the state in the Efficiency state courts, seeking money damages under the federal statute for the lost job. Is there any constitutional barrier to Meth's recovery?

Yes. Under Alden v. Maine, 527 U.S. 706 (1999), a constitutionally based "sovereign immunity" prevents the states from being required to entertain, in their own courts, a private damage suit based on violations of federal law. This immunity means that Congress can't do what it purported to do here—require Efficiency to allow itself to be sued for money damages in its own courts by a private party for a violation of the federal ADEA. NOTE: Alden says that this sovereign immunity is not specifically mentioned in the Constitution, but is tacitly recognized by the Tenth Amendment (see LSP/GEN 5). NOTE: Observe that when you combine Alden with the Court's recent decisions saying that Congress can't generally abrogate the Eleventh Amendment (e.g., Seminole Tribe), the states are largely immune from private damage suits for violations of federally created rights. So here, Meth can neither sue Efficiency in state court for age-discrimination damages despite the "protections" he supposedly has under the ADEA, nor sue the state in federal court for these damages. N.B.: Alden dealt with Congress' Article I powers. The federal government retains its powers under the Civil War Amendments to abrogate state sovereign immunity.

Albert "the Scar" Patchino is a big drug dealer in Miami. Congress levies a 60% excise tax on dealers in illegal narcotics. Only $200,000 is raised per year. Congress' principal purpose is to make dealing in illegal narcotics less profitable as well as more dangerous (because of the possibility of prosecution for tax evasion). Assuming Congress couldn't regulate drug dealers directly, is the tax valid?

Yes. When Congress taxes an activity it couldn't otherwise regulate, the tax will be valid if it either actually raises revenue or Congress intended that it do so. Here, the tax raises some revenue, so it will be upheld, even though Congress could not regulate drug dealers directly. U.S. v. Doremus, 249 U.S. 86 (1919).

Miss Brooks teaches at Shiny Apple State College. She is serving her fourth consecutive one-year contract. State law grants tenure after the completion of seven consecutive one-year contracts. Brooks is told she won't be rehired for a fifth year and is given no reasons why (which is proper under state rules). Could Brooks possibly establish a property right in her continued employment?

Yes; and, in doing so, she could claim her procedural due process rights were violated by her termination without a hearing. Procedural due process rights only attach when one is deprived of a property or liberty right. A legitimate claim to continued public employment requires a government-created expectancy, pursuant to applicable law (e.g., tenure). However, tenure can be de facto—that is, college rules, regulations, and practices could create a tenure system in fact. Perry v. Sindermann, 408 U.S. 593 (1972). Say, for instance, that the college president orally assured Brooks that she'd be rehired, and she reasonably relied on this promise. This would probably be enough to create a property right, such that due process rights would attach to her dismissal.

US v. Morrison

a United States Supreme Court decision which held that parts of the Violence Against Women Act of 1994 were unconstitutional because they exceeded congressional power under the Commerce Clause and under section 5 of the Fourteenth Amendment to the Constitution

Williamson v. Lee Optical

optician sought to have an oklahoma statute declared unconstitutional because the effect was to forbid an optician from fitting or duplicating lenses with a prescription from an ophthalmologist or optometrist. court admitted that the law might be wasteful but it was probably the legislatures intent for it to be that way. (now the court is hands off with economic regulation)


संबंधित स्टडी सेट्स

Comm 103: Chapter 1: A first look

View Set

General Real Property Law: Estates in Land

View Set

Computed Tomography for Technologists P2 (p 64 - 176)

View Set

Discuss MRSA, VRE, and C. Difficile

View Set

[TieuGia] PSM I Course_Question Bank_v1.0 - PART 03

View Set

CHEMISTRY:Naming and Writing Formulas For Acids and Bases

View Set